2017 (Paper 1) Flashcards

1
Q

Patient with collapse, P wave rate of 75, Broad qrs rate of 40

Options:

complete AV block

first degree HB

second degree HB

A

Complete AV block (Complete heart block)

Features:

  • syncope
  • heart failure
  • regular bradycardia (30-50 bpm)
  • wide pulse pressure
  • JVP: cannon waves in neck
  • variable intensity of S1

Types of heart block

  • First degree heart block
    • PR interval > 0.2 seconds
  • Second degree heart block
    • type 1 (Mobitz I, Wenckebach): progressive prolongation of the PR interval until a dropped beat occurs
    • type 2 (Mobitz II): PR interval is constant but the P wave is often not followed by a QRS complex
  • Third degree (complete) heart block
    • there is no association between the P waves and QRS complexes
How well did you know this?
1
Not at all
2
3
4
5
Perfectly
2
Q

Type 1 diabetic goes to an all night party, doesn’t eat and sleeps all day. 8pm presents to A&E with vomiting. pH 7.24

Options:

DKA

hypoglycaemia

A

DKA (Diabetic ketoacidosis)

may be a complication existing type 1 diabetes mellitus or be the first presentation, accounting for around 6% of cases. Whilst DKA remains a serious condition mortality rates have decreased from 8% to under 1% in the past 20 years.

The most common precipitating factors of DKA are infection, missed insulin doses and myocardial infarction

Features

  • abdominal pain
  • polyuria, polydipsia, dehydration
  • Kussmaul respiration (deep hyperventilation)
  • Acetone-smelling breath (‘pear drops’ smell)

Diagnostic criteria (joint british diabetes soc 2013)

  • Glucose >11 mmol or known DM
  • pH<7.3
  • bicarcb <15
  • Ketones >3 mmol/l or urine ketones ++ on dibpstick

Management

  • fluid replacement: most patients with DKA are deplete around 5-8 litres. Isotonic saline is used initially. Please see an example fluid regime below.
  • insulin: an intravenous infusion should be started at 0.1 unit/kg/hour. Once blood glucose is < 15 mmol/l an infusion of 5% dextrose should be started
  • correction of hypokalaemia

Complications of DKA (and it’s treatment)

  • gastric stasis
  • Thromboembolism
  • arrhythmia secondary to hyperkalaemia/isatrogenic hypokalaemia
  • iatrogenic due to incorrect fluid therapy:
    • cerebreal oedema, hypokalaemia, hypoglycaemia
  • ARDS
  • AKI
How well did you know this?
1
Not at all
2
3
4
5
Perfectly
3
Q

Student comes back to England from Nigeria with jaundice, anaemia and fever.

Options:

Falciparum

Hep A

Influenza A

Typhoid

Paratyphoid fever

A

Falicparum

Feature of severe malaria

  • schizonts on a blood film
  • parasitaemia > 2%
  • hypoglycaemia
  • acidosis
  • temperature > 39 °C
  • severe anaemia
  • complications:
    • cerebral malaria: seizures, coma
    • acute renal failure: blackwater fever, secondary to intravascular haemolysis, mechanism unknown
    • acute respiratory distress syndrome (ARDS)
    • hypoglycaemia
    • disseminated intravascular coagulation (DIC)

Uncomplicated falciparum malaria

  • strains resistant to chloroquine are prevalent in certain areas of Asia and Africa
  • the 2010 WHO guidelines recommend artemisinin-based combination therapies (ACTs) as first-line therapy
  • examples include artemether plus lumefantrine, artesunate plus amodiaquine, artesunate plus mefloquine, artesunate plus sulfadoxine-pyrimethamine, dihydroartemisinin plus piperaquine

Severe falciparum malaria

  • a parasite counts of more than 2% will usually need parenteral treatment irrespective of clinical state
  • intravenous artesunate is now recommended by WHO in preference to intravenous quinine
  • if parasite count > 10% then exchange transfusion should be considered
  • shock may indicate coexistent bacterial septicaemia - malaria rarely causes haemodynamic collapse
How well did you know this?
1
Not at all
2
3
4
5
Perfectly
4
Q

40 year old farmer has wheeze for a few weeks. Normal CXR. Diagnosis.

Options:

Asthma

Farmers lung

Aspergillosis

A

Allergic bronchopulmonary Aspergillosis

results from an allergy to Aspergillus spores. In the exam questions often give a history of bronchiectasis and eosinophilia.

Features

  • bronchoconstriction: wheeze, cough, dyspnoea
  • bronchiectasis (proximal)

Investigations

  • eosinophilia
  • flitting CXR changes
  • positive radioallergosorbent (RAST) test to Aspergillus
  • positive IgG precipitins (not as positive as in aspergilloma)
  • raised IgE

Management

  • steroids
  • itraconazole is sometimes introduced as a second line agent

Extrinsic allergic alverolitis e.g. farmers lung (spored of saccharopolyspora rectivirgula) is a hypersensitivity induce lung damage via a type III hypersensitivity immune complex mediated process. It presents acutely, c.4-8 hrs post exposure with SOB, dry cough, fever and may present chronically. Investigations: CXR (upper/mid zone fibrosis) bronchoalveolar lavage (lymphocytosis), blood (NO eosinophilia)

How well did you know this?
1
Not at all
2
3
4
5
Perfectly
5
Q

Anal abscess (described as perianal erythema and swelling) with fever and a lump. What do you do?

Options:

Give abx and review early

Incision and drainage

Oral flucloxacillin

A

Incision and drainage

Peri-anal abscess = perianal swelling & surrounding erythema

Treatment = I & D, leave the cavity open to heal by secondary intention

Management of benign proctology (see table and…):

  • Haemorroidal disease
    • The treatment of haemorroids is usually conservative. Acutely thrombosed haemorroids may be extremely painful. Treatment of this acute condition is usually conservative and consists of stool softeners, ice compressions and topical GTN or diltiazem to reduce sphincter spasm. Most cases managed with this approach will settle over the next 5-7 days. After this period there may be residual skin tags that merit surgical excision or indeed residual haemorroidal disease that may necessitate haemorroidectomy.
    • Patients with more chronic symptoms are managed according to the stage of their disease, small mild internal haemorroids causing little symptoms are best managed conservatively. More marked symptoms of bleeding and occasional prolapse, where the haemorroidal complex is largely internal may benefit from stapled haemorroidopexy. This procedure excises rectal tissue above the dentate line and disrupts the haemorroidal blood supply. At the same time the excisional component of the procedure means that the haemorroids are less prone to prolapse. Adverse effects of this procedure include urgency, which can affect up to 40% of patients (but settles over 6-12 months) and recurrence. The procedure does not address skin tags and therefore this procedure is unsuitable if this is the dominant symptom.
    • Large haemorroids with a substantial external component may be best managed with a Milligan Morgan style conventional haemorroidectomy. In this procedure three haemorroidal cushions are excised, together with their vascular pedicle. Excision of excessive volumes of tissue may result in anal stenosis. The procedure is quite painful and most surgeons prescribe metronidazole post operatively as it decreases post operative pain.
  • Fissure in ano
    • Probably the most efficient and definitive treatment for fissure in ano is lateral internal sphincterotomy. The treatment is permanent and nearly all patients will recover. Up to 30% will develop incontinence to flatus. There are justifiable concerns about using this procedure in females as pregnancy and pelvic floor damage together with a sphincterotomy may result in faecal incontinence. The usual first line therapy is relaxation of the internal sphincter with either GTN or diltiazem (the latter being better tolerated) applied topically for 6 weeks. Treatment failures with topical therapy will usually go on to have treatment with botulinum toxin. This leads to more permanent changes in the sphincter and this may facilitate healing.
    • Typical fissures usually present in the posterior midline, multiple or unusually located fissures should prompt a search for an underlying cause such as inflammatory bowel disease or internal prolapse.
    • Refractory cases where the above treatments have failed may be considered for advancement flaps.
  • Fistula in ano
    • The most effective treatment for fistula is laying it open (fistulotomy). When the fistula is below the sphincter and uncomplicated, this is a reasonable option. Sphincter involvement and complex underlying disease should be assessed both surgically and ideally with imaging (either MRI or endoanal USS). Surgery is then usually staged, in the first instance a draining seton suture may be inserted. This avoids the development of recurrent sepsis and may allow resolution. In patients with Crohns disease the seton should be left in situ long term and the patient managed medically, as in these cases attempts at complex surgical repair nearly always fail. Fistulas not associated with IBD may be managed by advancement flaps, instillation of plugs and glue is generally unsuccessful. A newer technique of ligation of intersphincteric tract (LIFT procedure) is reported to have good results in selected centres.
How well did you know this?
1
Not at all
2
3
4
5
Perfectly
6
Q

Pain on passive dorsiflexion Compartment syndrome young guy fractured tibia playing football.

Options:

4 compartment fasciotomy within 6h

Review in 12hrs

Send home

A

4 compartment fasciotomy within 6h

Compartment syndrome

This is a particular complication that may occur following fractures (or following ischaemia re-perfusion injury in vascular patients). It is characterised by raised pressure within a closed anatomical space.

The raised pressure within the compartment will eventually compromise tissue perfusion resulting in necrosis. The two main fractures carrying this complication include supracondylar fractures and tibial shaft injuries.

Symptoms and signs

  • Pain, especially on movement (even passive)
  • Parasthesiae
  • Pallor may be present
  • Arterial pulsation may still be felt as the necrosis occurs as a result of microvascular compromise
  • Paralysis of the muscle group may occur

Diagnosis: Is made by measurement of intracompartmental pressure measurements. Pressures in excess of 20mmHg are abnormal and >40mmHg is diagnostic.

Treatment

  • This is essentially prompt and extensive fasciotomies
  • In the lower limb the deep muscles may be inadequately decompressed by the inexperienced operator when smaller incisions are performed
  • Myoglobinuria may occur following fasciotomy and result in renal failure and for this reason these patients require aggressive IV fluids
  • Where muscle groups are frankly necrotic at fasciotomy they should be debrided and amputation may have to be considered
  • Death of muscle groups may occur within 4-6 hours
How well did you know this?
1
Not at all
2
3
4
5
Perfectly
7
Q

30something man with joint pain - sacroiliitis and distal interphalangeal joint pain?

Options:

Ankylosing spondylitis

Psoriatic arthritis

Rheumatoid arthritis.

A

Psoriatic arthropathy correlates poorly with cutaneous psoriasis and often precedes the development of skin lesions. Around 10-20% percent of patients with skin lesions develop an arthropathy with males and females being equally affected

Types*

  • rheumatoid-like polyarthritis: (30-40%, most common type)
  • asymmetrical oligoarthritis: typically affects hands and feet (20-30%)
  • sacroilitis
  • DIP joint disease (10%)
  • arthritis mutilans (severe deformity fingers/hand, ‘telescoping fingers’)

Management:

treat as rheumatoid arthritis

but better prognosis

How well did you know this?
1
Not at all
2
3
4
5
Perfectly
8
Q

Transfusion reaction - fever, other obs normal, transfusion stopped. What next?

Options:

Check bag against patient details

Start transfusion again

Give chlorpheniramine

A

Check bag against patient details

Febrile non-haemolytic transfusion reaction

  • Signs and symptoms: Fever, chills, headache, malaise, flushing
  • Mechanism: host abs vs donor MHC antigens or due to cytokines from leukocytes in donor blood
  • Treatment: discontinue transfusion, fever resolves in 15-30 mins without specific treatment. Paracetamol may be used
  • Other: most common transusion reaction, can be prevented with leukocyte filters or irradiation

Haemolytic transfusion reaction

  • Signs and symptoms: Fever, chills, pain at the infusion site, dark urine, nausea, shock
  • Mechanism: ABO incompatibility with host abs vs antigens on donor RBCs
  • Treatment: discontinue transfusion, administer fluids
  • Other: most SEVERE reaction

Allergic transfusion reaction

  • Signs and symptoms: Uritcaria, pruritus
  • Mechanism: allergic reaction to plasma proteins in transfused blood
  • Treatment: DOES NOT require discontinuing transfusion, symptomatic treatment with antihistamines
  • Other: can be prevented with anti-histamine pre-treatment

Anaphylactic transfusion reaction

  • Signs and symptoms: Uritcaria, angioedema, wheezing, laryngeal oedema, abdo pain, hypotension, shock
  • Mechanism: Host abs vs IgA antibodies in the donor plasma
  • Treatment: discontinue transfusion, administer adrenaline
  • Other: Seen in pts with IgA deficiency, can be prevented by administering washed IgA decifient products

Transfusion relatied acute lung injury (TRALI)

  • Signs and symptoms: dyspnoea, hypoxaemia, bilateral chest infiltrates
  • Mechanism: donor Abs to MHC class I/II/Human neutrophil antigens. Activated neutrophils cause endothelial damage
  • Treatment: discontinue transfusion, provide airway support
  • Other: most common cause of transfusion related DEATH

Transfusion associated circulatory overload

  • Signs and symptoms: dyspnoea, pulmonary oedema, HTN, peripheral oedema
  • Mechanism: rapid volume expansion.
  • Treatment: supportive, diuretics can be used
  • Other: seen in elderly pts with HF/anaemia, can be prevented with slower transfusions and diuretics
How well did you know this?
1
Not at all
2
3
4
5
Perfectly
9
Q

Patient on warfarin having nasal polypectomy - what do you do to the warfarin.

Options:

Admit patient two days pre-op and start heparin

Change to aspirin after op to reduce risk of bleed

Measure aptt

Stop warfarin on the day of surgery

A

Admit patient two days pre-op and start heparin

the newer oral anticoagulants (dabigatran, rivaroxaban, apixaban) have shorter half-lives and faster onset of action compared to warfarin and bridging is generally not required

aPTT, intrinsic pathway (12), used to monitor hepatin

PT, extrinsic pathway (7), used to monitor warfarin

Warfarin: inhibits the reductase enzyme responsible for the active form of vitamin K therefore inhibits the synthesis of factors: 2, 7, 9, 10, C, S, Z

Nasal polypectomy is a procedue with a High Risk Bleeding (greater than 1.5% or in vulnerable areas).

Holding warfarin before surgery:

  • For patients whose INR is between 2.0 and 3.0, discontinue warfarin 5 days prior to surgery (last dose given 6 days before surgery) and allow the INR to spontaneously fall. Warfarin should be withheld for a longer period of time if the INR is normally maintained above 3.0.
  • The INR should be measured the day prior to surgery. Vitamin K may be administered if the INR is deemed excessive.

Bridging with IV unfractionated heparin before surgery:

  • After discontinuation of warfarin, patients should be admitted to hospital and started on IV unfractionated heparin in therapeutic doses. Since therapeutic oral anticoagulation will remain therapeutic for at least a day after the last warfarin dose, patients can be admitted on the second day after their last dose of warfarin.
  • IV heparin should be discontinued 4 to 6 hours prior to surgery.

Restarting warfarin after surgery

  • Post-operatively, warfarin should be resumed when the patient is able to take medications by mouth and after the epidural catheter has been removed (if neuraxial analgesia has been used).
  • Bridging with IV unfractionated heparin after surgery: Full dose (therapeutic dose) IV unfractionated heparin should be started no sooner than 24 hours after major surgery when there is adequate post-op hemostasis. If there is evidence of surgical bleeding or if the patient is at high risk of bleeding, it should be delayed further. It should also be delayed while the epidural catheter is insitu (if neuraxial analgesia has been used). In situations where therapeutic dose IV unfractionated heparin is deferred beyond 24 hours, the administration of prophylactic dose LMWH can be considered sooner (as early as the evening of the day of surgery). IV heparin may be started sooner if the surgery or procedure is of a minor nature and the risk of bleeding is low. Heparin should be continued until the INR is therapeutic.
How well did you know this?
1
Not at all
2
3
4
5
Perfectly
10
Q

Breast Ca - which is the biggest risk factor in this patient?

Options

Obesity

Smoking

Multiple pregnancies

Breastfeeding

Late menarche

A

Obesity

  • Incidence: most common cancer in women, lifetime risk 1 in 8
  • Age: 75-80yrs, (younger in Afro-Caribbean’s). Sex: 99% in women.
  • Risk factors:
    • Susceptibility genes (12%) – BRCA1/BRCA2, also increased risk of ovarian, prostate and pancreatic malignancy. BRCA mutations cause a lifetime risk of invasive breast carcinoma of up to 85%.
    • Hormone exposure
      • Early menarche
      • Late menopause
      • Late 1st live birth (pregnancy → terminal differentiation of milk-producing luminal cells, removing these from pool of potential cancer precursors)
      • OCP/HRT
    • Advancing age
    • Family history
    • Race (Caucasian>Afro-Caribbean>Asian>Hispanic)
    • Obesity, tobacco, alcohol
  • Presentation: hard fixed lump, Paget’s disease, peau d’orange, nipple retraction.
  • Screening: 47 to 73yr old women invited every 3 years for mammography (looks for abnormal areas of calcification or a mass within the breast)
How well did you know this?
1
Not at all
2
3
4
5
Perfectly
11
Q

Legionella - which Abx?

Options:

Ciprofloxacin

Clarithromycin

Cefotaxime

Cefalexin

A

Clarithromycin

Legionella

Legionnaire’s disease is caused by the intracellular bacterium Legionella pneumophilia. It is typically colonizes water tanks and hence questions may hint at air-conditioning systems or foreign holidays. Person-to-person transmission is not seen

Features

  • flu-like symptoms including fever (present in > 95% of patients)
  • dry cough
  • relative bradycardia
  • confusion
  • lymphopaenia
  • hyponatraemia
  • deranged liver function tests
  • pleural effusion: seen in around 30% of patients

Diagnosis: urinary antigen
Management: treat with erythromycin

How well did you know this?
1
Not at all
2
3
4
5
Perfectly
12
Q

Which drug is commonly co-prescribed with morphine?

Options:

Aspirin

Hyoscine

Co-danthramer

Loperamide

A

Co-danthramer

Co-danthramer: Constipation is a common cause of distress and is almost invariable after administration of an opioid analgesic. It should be prevented if possible by the regular administration of laxatives; a faecal softener with a peristaltic stimulant (e.g. co-danthramer) or lactulose solution with a senna preparation should be used. Methylnaltrexone bromide is licensed for the treatment of opioid-induced constipation.

Hyoscine: Bowel colic and excessive respiratory secretions may be reduced by a subcutaneous injection of hyoscine hydrobromide, hyoscine butylbromide, or glycopyrronium bromide. These antimuscarinics are generally given every 4 hours when required, but hourly use is occasionally necessary, particularly in excessive respiratory secretions. If symptoms persist, they can be given regularly via a continuous infusion device. Care is required to avoid the discomfort of dry mouth.

Loperamide: The pain of bowel colic may be reduced by loperamide hydrochloride. Hyoscine hydrobromide may also be helpful, given sublingually as Kwells ® tablets. Subcutaneous injections of hyoscine butylbromide, hyoscine hydrobromide, and glycopyrronium bromide can also be used to treat bowel colic.

Gastric distension pain due to pressure on the stomach may be helped by a preparation incorporating an antacid with an antiflatulent and a prokinetic such as domperidone before meals.

How well did you know this?
1
Not at all
2
3
4
5
Perfectly
13
Q

Mid-diastolic murmur

A

Mitral stenosis

Ejection systolic

  • aortic stenosis
  • pulmonary stenosis, HOCM
  • ASD, Fallot’s

Holosystolic (pansystolic)

  • mitral/tricuspid regurgitation (high-pitched and ‘blowing’ in character)
  • VSD (‘harsh’ in character)

Late systolic

  • mitral valve prolapse
  • coarctation of aorta

Early diastolic

  • aortic regurgitation (high-pitched and ‘blowing’ in character)
  • Graham-Steel murmur (pulmonary regurgitation, again high-pitched and ‘blowing’ in character)

Mid-late diastolic

  • mitral stenosis (‘rumbling’ in character)
  • Austin-Flint murmur (severe aortic regurgitation, again is ‘rumbling’ in character)

Continuous machine-like mumur

  • patent ductus arteriosus
How well did you know this?
1
Not at all
2
3
4
5
Perfectly
14
Q

Pericarditis murmur qs - Patient has widespread ST elevation, which sound heard on ausc?

A

Scratch (pericardial friction rub)

Pericarditis is one of the differentials of any patient presenting with chest pain.
Features

  • chest pain: may be pleuritic. Is often relieved by sitting forwards
  • other symptoms include non-productive cough, dyspnoea and flu-like symptoms
  • pericardial rub
  • tachypnoea
  • tachycardia

Causes

  • viral infections (Coxsackie)
  • tuberculosis
  • uraemia (causes ‘fibrinous’ pericarditis)
  • trauma
  • post-myocardial infarction, Dressler’s syndrome
  • connective tissue disease
  • hypothyroidism

ECG changes

  • widespread ‘saddle-shaped’ ST elevation
  • PR depression: most specific ECG marker for pericarditis

Management: if acuse, analgesia ibuprofen PI and Rx cause, consider steroids and immunosuppresion

How well did you know this?
1
Not at all
2
3
4
5
Perfectly
15
Q

Which antibodies are most specific/raised in SLE

Options:

dsDNA

anti-cardiolipin

anti-Smith

Anti-U1 RNP

A

Anti-Smith

Immunology

  • 99% are ANA positive (sensitive)
  • 20% are rheumatoid factor positive
  • anti-dsDNA: highly specific (> 99%), but less sensitive (70%)
  • anti-Smith: most specific (> 99%), sensitivity (30%)
  • also: anti-U1 RNP, SS-A (anti-Ro) and SS-B (anti-La)

Monitoring

  • ESR: during active disease the CRP is characteristically normal - a raised CRP may indicate underlying infection
  • complement levels (C3, C4) are low during active disease (formation of complexes leads to consumption of complement)
  • anti-dsDNA titres can be used for disease monitoring (but note not present in all patients)
How well did you know this?
1
Not at all
2
3
4
5
Perfectly
16
Q

EDM Aortic regurg murmur with sudden onset chest pain going to back.

Diagnosis?

A

Aortic dissection

Aortic dissection is a rare but serious cause of chest pain.

Associations

  • hypertension
  • trauma
  • bicuspid aortic valve
  • collagens: Marfan’s syndrome, Ehlers-Danlos syndrome
  • Turner’s and Noonan’s syndrome
  • pregnancy
  • syphilis

Features:

  • chest pain: typically severe, radiates through to the back and ‘tearing’ in nature
  • aortic regurgitation
  • hypertension
  • other features may result from the involvement of specific arteries. For example:
    • coronary arteries → angina
    • spinal arteries → paraplegia
    • distal aorta → limb ischaemia

Classification

  • Stanford classification
    • type A - ascending aorta, 2/3 of cases
    • type B - descending aorta, distal to left subclavian origin, 1/3 of cases
  • DeBakey classification
    • type I - originates in ascending aorta, propagates to at least the aortic arch and possibly beyond it distally
    • type II - originates in and is confined to the ascending aorta
    • type III - originates in descending aorta, rarely extends proximally but will extend distally

Management

  • Type A: surgical management, but blood pressure should be controlled to a target systolic of 100-120 mmHg whilst awaiting intervention
  • Type B*: conservative management = bed rest + reduce blood pressure IV labetalol to prevent progression

Complications

  • Complications of backward tear
    • aortic incompetence/regurgitation
    • MI: inferior pattern often seen due to right coronary involvement
  • Complications of forward tear
    • unequal arm pulses and BP
    • stroke
    • renal failure
How well did you know this?
1
Not at all
2
3
4
5
Perfectly
17
Q

Patient with symptoms of aortic dissection - AR etc, what test for confirming diagnosis?

Otions:

Contrast CT chest

CXR

USS

A

Contrast CT chest

Because of the varying symptoms and signs of aortic dissection depending on the initial intimal tear and the extent of the dissection, the proper diagnosis is sometimes difficult to make.

While taking a good history from the individual may be strongly suggestive of an aortic dissection, the diagnosis cannot always be made by history and physical signs alone. Often, the diagnosis is made by visualization of the intimal flap on a diagnostic imaging test. Common tests used to diagnose an aortic dissection include a CT scan of the chest with iodinated contrast material and a transesophageal echocardiogram.

Investigations:

  • Bloods: FBC, cross-match 10 units of blood, U&E (renal function), clotting.
  • CXR: Widened mediastinum, localized bulge in the aortic arch.
  • ECG: Often normal. Signs of left ventricular hypertrophy or inferior MI if dissection compromises the ostia of the right coronary artery.
  • CT-thorax: False lumen of dissection can be visualized.
  • Echocardiography: Transoesophageal is highly specific.
  • Cardiac catheterization and aortography.
How well did you know this?
1
Not at all
2
3
4
5
Perfectly
18
Q

Lady with yellow eyes and high reticulocytes.

Options:

Alcoholic hepatitis

Haemolysis

Viral Hepatitis

A

Haemolysis

How well did you know this?
1
Not at all
2
3
4
5
Perfectly
19
Q

70yo lady falls in road onto outstretched hand and gets fracture. What test should GP follow up with?

Options

Vit d levels

DEXA

Bone profile

A

DEXA

Indications for a DEXA scan:

  • a patient with a history of a low trauma fracture e.g. spine, hip, distal forearm
  • an incidental X-ray finding of osteopenia or vertebral collapse or in the investigation of thoracic kyphosis or loss of height
  • if there is maternal history of hip fracture
  • if the patient has a low body mass index (BMI < 19 kg/m^2)
  • if the patient has received corticosteroid treatment (prednisolone 7.5 mg or more) for greater than three months
  • oestrogen deficiency:
    • in a patient with premature menopause (< 45 years of age)
    • primary hypogonadism
    • secondary amenorrhoea (which lasts more than six months) e.g. secondary to GnRH anologues
  • where there are conditions that predispose to secondary osteoporosis including:
    • malabsorption syndromes e.g. coeliac disease, inflammatory bowel disease
    • long-term treatment with anticonvulsants
    • organ transplantation
    • eating disorders
    • chronic renal failure
    • thyrotoxicosis
    • primary hyperparathyroidism
    • Cushing’s syndrome
    • male hypogonadism
    • prolonged immobilisation
How well did you know this?
1
Not at all
2
3
4
5
Perfectly
20
Q

Patient on warfarin for AF has INR of 3.3, falls and found on CT to have intracerebral haematoma - warfarin stopped and been given Vit K, what next?

Options

Prothrombin complex

Vit K again in 12 hours

Mannitol

A

Prothrombin Complex

For Major bleeding, including intracranial haemorrhage: Stop warfarin. Give prothrombin complex concentrate. If unavailable, give FFP. Also give vitamin K IV.

How well did you know this?
1
Not at all
2
3
4
5
Perfectly
21
Q

Patient has central chest pain, which ECG criteria would be indication for thrombolysis?

Options:

New RBBB

ST elevation in leads II, III and aVF

T wave inversion in aVR

ST depression in lead v1-4

A

ST elevation in leads II, III and aVF

  1. New RBBB: could be PE with RV dysfunction
  2. ST elevation in leads II, III and aVF: inferior STEMI
  3. T wave inversion in aVR: normal in aVR and V1
  4. ST depression in lead v1-4: ischaemia/NSTEMI (if raise troponin)

With regards to thrombolysis:

Primary percutaneous coronary intervention (PCI) has emerged as the gold-standard treatment for STEMI but is not available in all centres. Thrombolysis should be performed in patients without access to primary PCI

  • Tissue plasminogen activator (tPA) has been shown to offer clear mortality benefits over streptokinase
  • Tenecteplase is easier to administer and has been shown to have non-inferior efficacy to alteplase with a similar adverse effect profile
  • An ECG should be performed 90 minutes following thrombolysis to assess whether there has been a greater than 50% resolution in the ST elevation
    • if there has not been adequate resolution then rescue PCI is superior to repeat thrombolysis
    • for patients successfully treated with thrombolysis PCI has been shown to be beneficial. The optimal timing of this is still under investigation

RBBB:

  • Right ventricular hypertrophy / cor pulmonale
  • Pulmonary embolus
  • Ischaemic heart disease
  • Rheumatic heart disease
  • Myocarditis or cardiomyopathy
  • Degenerative disease of the conduction system
  • Congenital heart disease (e.g. atrial septal defect)
How well did you know this?
1
Not at all
2
3
4
5
Perfectly
22
Q

Male married 30 years, sex with only his wife, pain in testicle and epididymis. Organism?

Options

Chlamydia trachomatis

E.Coli

Neisseria gonorrhoea

A

E.coli

  • Elderly, and unable to distinguish between two sti*
  • bacterial infections are the most common aetiology for epididymitis*
  • in men <= 35 years of age, ascending infection from the urethra by sexually transmittable pathogens, namely Chlamydia trachomatis and Neisseria gonorrhoeae, are aetiologically responsible
  • in older men with a history of bladder outlet disturbances
    • Enterobacteriaceae, particularly Escherichia coli, represent the dominant pathogens
  • tuberculous epididymitis caused by mycobacteria must be considered as a differential diagnosis

Epididymo-orchitis describes an infection of the epididymis +/- testes resulting in pain and swelling. It is most commonly caused by local spread of infections from the genital tract (such as Chlamydia trachomatis and Neisseria gonorrhoeae) or the bladder.

The most important differential diagnosis is testicular torsion. This needs to be excluded urgently to prevent ischaemia of the testicle.
Features

  • unilateral testicular pain and swelling
  • urethral discharge may be present, but urethritis is often asymptomatic
  • factors suggesting testicular torsion include patients < 20 years, severe pain and an acute onset

Management

  • the British Association for Sexual Health and HIV (BASHH) produced guidelines in 2010
  • if the organism is unknown BASHH recommend: ceftriaxone 500mg intramuscularly single dose, plus doxycycline 100mg by mouth twice daily for 10-14 days
  • further investigations following treatment are recommended to exclude any underlying structural abnormalities

Chlamydia

is the most prevalent sexually transmitted infection in the UK and is caused by Chlamydia trachomatis, an obligate intracellular pathogen. Approximately 1 in 10 young women in the UK have Chlamydia. The incubation period is around 7-21 days, although it should be remembered a large percentage of cases are asymptomatic
Features

  • asymptomatic in around 70% of women and 50% of men
  • women: cervicitis (discharge, bleeding), dysuria
  • men: urethral discharge, dysuria

Potential complications

  • epididymitis
  • pelvic inflammatory disease
  • endometritis
  • increased incidence of ectopic pregnancies
  • infertility
  • reactive arthritis
  • perihepatitis (Fitz-Hugh-Curtis syndrome)

Investigation

  • traditional cell culture is no longer widely used
  • nuclear acid amplification tests (NAATs) are now rapidly emerging as the investigation of choice
  • urine (first void urine sample), vulvovaginal swab or cervical swab may be tested using the NAAT technique

Screening

  • in England the National Chlamydia Screening Programme is open to all men and women aged 15-24 years
  • the 2009 SIGN guidelines support this approach, suggesting screening all sexually active patients aged 15-24 years
  • relies heavily on opportunistic testing
How well did you know this?
1
Not at all
2
3
4
5
Perfectly
23
Q

45 year old with intracapsular #NOF otherwise well (not given Garden classification) - management?

Options

Hemiarthroplasty

Dynamic hip screw

Cannulated screws

A

Cannulated screws

Garden 1 and 2
Cannulated screw
Garden 3 and 4
Ain’t hemi anymore
Between greater lesser
DHS is better
If below the neck should fail
Inter medullary nail

NOT DHS- ‘If you can get adequate fixation with the least amount of metal then it’s ideal, specifically in a patient in which you want to maximise bone salvage and minimise instrumentation’

NOF The hip is a common site of fracture especially in osteoporotic, elderly females. The blood supply to the femoral head runs up the neck and thus avascular necrosis is a risk in displaced fractures.

Features:

  • pain
  • shortened and externally rotated leg

The Garden system:

  • Type I: Stable fracture with impaction in valgus.
  • Type II: Complete fracture but undisplaced.
  • Type III: Displaced fracture, usually rotated and angulated, but still has bony contact.
  • Type IV: Complete bony disruption.
How well did you know this?
1
Not at all
2
3
4
5
Perfectly
24
Q

Patient has cough, apical cavitating lesions, haemoptysis, firm LN in neck. What test?

Options:

Auramine stain of sputum

Sputum culture and sensitivity for AAFB

LN biopsy

A

Auramine stain of sputum -> next step in management

Sputum culture and sensitivity for AAFB -> Confirm diagnosis and treatment specifically

Ziehl-Nielsen or auramine staining of a sputum smear may demonstrate the presence of acid-fast bacilli in vitro culture of the sputum may take 4 to 7 weeks to provide a result; a further 3 weeks is required to identify drug sensitivity

Diagnosis Latent TB: Do a Mantoux test. If +ve (or non-reliable) consider interferongamma testing
Active TB: If CXR suggests TB, take sputum samples (≥3, with one early morning
sample, before starting treatment if possible) and send for MC&S for AFB (acid-fast
bacilli resist acid on Ziehl–Neelsen (ZN) staining). If spontaneously produced sputum cannot be obtained, bronchoscopy and lavage may be needed.
Active non-respiratory TB: Try hard to get samples: sputum, pleura & pleural fl uid,
urine, pus, ascites, peritoneum, bone marrow or CSF. Send surgical samples for
culture. Microbiologist should routinely do TB culture on these, even if it is not requested. All patients with non-respiratory TB should have a CXR to find coexisting respiratory TB. Incubate cultures for up to 12wks on Lowenstein–Jensen medium.

PCR: Allows rapid identifi cation of rifampicin (and likely multidrug) resistance.
Histology: The hallmark is the presence of caseating granulomata.
CXR signs: Consolidation, cavitation, fi brosis, and calcifi cation.

Immunological evidence of TB may be helpful:

  • Tuberculin skin test: TB antigen is
  • injected intradermally and the cell-mediated response at 48–72h is recorded. A +ve test indicates immunity. It may also indicate previous exposure or BCG. A strong+ve test probably means active TB. False –ve tests occur in immuno suppression (miliary TB, sarcoid, AIDS, lymphoma).
  • Quantiferon TB Gold® and T-spot-TB® tests measure the delayed hypersens itivity reaction developed after contact with M. tuberculosis;they use specifi c, complex M. tuberculosis antigens and are better than older Mantoux tests, which rely on reactions to serial dilutions of TB antigen.
How well did you know this?
1
Not at all
2
3
4
5
Perfectly
25
Q

Lady with palmar erythema, raised ALT, raised bilirubin, strongly positive ANA

Options

AI hepatitis

SLE

Drug induced cholestasis

PBC

A

Autoimmune hepatitis

… is condition of unknown aetiology which is most commonly seen in young females. Recognised associations include other autoimmune disorders, hypergammaglobulinaemia and HLA B8, DR3. Three types of autoimmune hepatitis have been characterised according to the types of circulating antibodies present

  • Type I
    • Anti-nuclear antibodies (ANA) and/or anti-smooth muscle antibodies (SMA)
    • Affects both adults and children
  • Type II
    • ​Anti-liver/kidney microsomal type 1 antibodies (LKM1)
    • Affects children only
  • Soluble liver-kidney antigen
    • Affects adults in middle-age

Features

  • may present with signs of chronic liver disease
  • acute hepatitis: fever, jaundice etc (only 25% present in this way)
  • amenorrhoea (common)
  • ANA/SMA/LKM1 antibodies, raised IgG levels
  • liver biopsy: inflammation extending beyond limiting plate ‘piecemeal necrosis’, bridging necrosis

Management

  • steroids, other immunosuppressants e.g. azathioprine
  • liver transplantation

Chronic liver disease:

  • General
    • Jaundice
    • Ascites
    • Cachexia
    • Tattoos and track marks
    • Pigmentation
  • Hands
    • Clubbing (esp. in PBC)
    • Leukonychia
    • Terry’s nails (white proximally, red distally)
    • Palmer erythema
    • Dupuytren’s contracture
  • Face
    • Pallor: ACD
    • Xanthelasma: PBC
    • Keiser-Fleischer rings
    • Parotid enlargement (esp. c¯ EtOH)
  • Trunk
    • Spider naevi
    • Gynaecomastia
    • Loss of 2O sexual hair
  • Ankles
    • Peripheral oedema
  • Abdomen
    • Inspection
      • Distension ± Para- / umbilical hernia
      • Dilated veins
      • Drain scars
    • Palpation
      • ± hepatomegaly
      • ± splenomegaly
      • Shifting dullness
How well did you know this?
1
Not at all
2
3
4
5
Perfectly
26
Q

Haematemesis and abdo pain, BP 120/80, HR >100 What do you give IV whilst waiting for endoscopy?

Options

IV crystalloid

O-ve blood

IV terlipressin

IV PPI

A

IV crystalloid

PPI not given before endoscopy, O-ve blood – no evidence of anaemia and acute resuscitation is with crystalloids to maintain BP first, Terlipression only if suspected/confirmed variceal bleed

ABCDE: CIRCULATION!

  • Resuscitate:
    • Head down
  • Airway: Early control of airway is vital (e.g. Drowsy patient with liver failure) -> protect the airway
  • Breathing: 100% O2
  • Circulation:
    • 2x 14G cannulae:
    • Bloods: cross match blood, check FBC, LFTs (ETOH abuse), U+E (protein meal) and Clotting and ABG (lactate + Hb) and glucose
    • IV crystalloid infusion up to 1L (avoid dilution)
  • D:
    • blood glucose, GCS, Temperature, expose (PR; maleana)

Other:

  • Major H’gge protocol: on-going bleeding and haemodynamic instability are likely to require O negative blood pending cross matched blood. ​keep Hb >10
  • Upper GI endoscopy (scoring systems)
    • all within 24 hours of admission.
    • If unstable = immediately after resuscitation or in tandem with it.

Specific:

  • Varices:
    • terlipressin prior to endoscopy​ (splanchnic vasopressor)
    • Prophylactic ABx e.g. ciprofloxacin 1g/24h
    • Varices should be banded or subjected to sclerotherapy, adrenaline coagulation
    • If this is not possible owing to active bleeding then a Sengaksten- Blakemore tube(or Minnesota tube) should be inserted. This should be done with care; gastric balloon should be inflated first and oesophageal balloon second. Remember the balloon with need deflating after 12 hours (ideally sooner) to prevent necrosis.
    • Portal pressure should be lowered by combination of medical therapy +/- TIPSS.
  • Oesophagitis/gastritis:
    • Patients with erosive oesophagitis / gastritis should receive a proton pump inhibitor.
    • Patients with diffuse erosive gastritis who cannot be managed endoscopically and continue to bleed may require gastrectomy
    • Haemostasis of vessel or ulcer
    • Identifiable bleeding points should receive combination therapy of injection of: adrenaline and either a thermal/laser coagulation, fibrin glu, endoclips.
    • All who have received intervention should receive a continuous infusion of a proton pump inhibitor (IV omeprazole for 72 hours) to reduce the re-bleeding rate.
    • Bleeding ulcers that cannot be controlled endoscopically may require: Gastric ulcer
    • Under-running of the bleeding site
    • Partial gastrectomy-antral ulcer
    • Partial gastrectomy or under running the ulcer- lesser curve ulcer (involving left gastric artery)
    • Total gastrectomy if bleeding persists
  • Duodenal ulcer:
    • Laparotomy, duodenotomy and under running of the ulcer. If bleeding is brisk then the ulcer is almost always posteriorly sited and will have invaded the gastroduodenal artery. Large bites using 0 Vicryl are taken above and below the ulcer base to occlude the vessel. The duodenotomy should be longitudinal but closed transversely to avoid stenosis.
  • Mallory Weiss: tears will typically resolve spontaneously
How well did you know this?
1
Not at all
2
3
4
5
Perfectly
27
Q

Alcoholic with diplopia on looking laterally both sides, nystagmus, what vitamin to give him?

A

Thiamine

….is a water soluble vitamin of the B complex group. One of it’s phosphate derivates, thiamine pyrophosphate (TPP), is an important coenzyme.
Thiamine is therefore important in the catabolism of sugars and aminoacids. The clinical consequences of thiamine deficiency are therefore seen first in highly aerobic tissues such as the brain (Wenicke-Korsakoff syndrome) and the heart (wet beriberi).
Causes of thiamine deficiency:

  • alcohol excess
  • malnutrition

Conditions associated with thiamine deficiency:

  • Wernicke’s encephalopathy: nystagmus, ophthalmoplegia and ataxia
  • Korsakoff’s syndrome: amnesia, confabulation
  • dry beriberi: peripheral neuropathy
  • wet beriberi: dilated cardiomyopathy

Wernicke’s encephalopathy

…is a neuropsychiatric disorder caused by thiamine deficiency which is most commonly seen in alcoholics. Rarer causes include: persistent vomiting, stomach cancer, dietary deficiency. A classic triad of ophthalmoplegia/nystagmus, ataxia and confusion may occur. In Wernicke’s encephalopathy petechial haemorrhages occur in a variety of structures in the brain including the mamillary bodies and ventricle walls
Features

  • nystagmus (the most common ocular sign)
  • ophthalmoplegia
  • ataxia
  • confusion, altered GCS
  • peripheral sensory neuropathy

Investigations

  • decreased red cell transketolase
  • MRI

Treatment is with urgent replacement of thiamine
Relationship with Korsakoff syndrome

  • If not treated Korsakoff’s syndrome may develop as well. This is termed Wernicke-Korsakoff syndrome and is characterised by the addition of antero- and retrograde amnesia and confabulation in addition to the above symptoms.
How well did you know this?
1
Not at all
2
3
4
5
Perfectly
28
Q

Diabetic with weakness in all four limbs started 3 days ago, weakness worse proximally cf distally in LL, no change to sensation in LL, UL has some pins and needles, bilateral eyelid weakness. Loss of reflex and plantar response.

Options:

GBS

Myasthenia

Diabetic neuropathy

MS

A

GBS

Guillain-Barre syndrome describes an immune mediated demyelination of the peripheral nervous system often triggered by an infection classically Campylobacter jejuni)

Pathogenesis

  • cross reaction of antibodies with gangliosides in the peripheral nervous system
  • correlation between anti-ganglioside antibody (e.g. anti-GM1) and clinical features has been demonstrated
  • anti-GM1 antibodies in 25% of patients

Features

The characteristic features of Guillain-Barre syndrome is progressive weakness of all four limbs. The weakness is classically ascending i.e. the lower extremities are affected first, however it tends to affect proximal muscles earlier than the distal ones. Sensory symptoms tend to be mild (e.g. distal paraesthesia) with very few sensory signs. Some patients experience back pain in the initial stages of the illness.
Other features

  • areflexia
  • cranial nerve involvement e.g. diplopia
  • autonomic involvement: e.g. urinary retention
  • Less common findings
    • papilloedema: thought to be secondary to reduced CSF resorption

Miller Fisher syndrome

  • variant of Guillain-Barre syndrome
  • associated with ophthalmoplegia, areflexia and ataxia. The eye muscles are typically affected first
  • usually presents as a descending paralysis rather than ascending as seen in other forms of Guillain-Barre syndrome
  • anti-GQ1b antibodies are present in 90% of cases
How well did you know this?
1
Not at all
2
3
4
5
Perfectly
29
Q

Alcoholic with multiple bruises on legs, has bloods and clotting which show (Low bili), low platelets, low fibrinogen, raised APTT, raised PT - cause of his clotting abnormalities?

Options

Liver disease

DIC

A

DIC

  • DIC = low fibrinogen, low platelets (consumption), petechiae, raised BT/PT/aPTT, signs of cause (sepsis, malignancy, trauma, obstetric)
  • Liver disease = clotting factors deficiency, low fibrinogen synthesis, chronic history, abnormality of platelet function, but _no quantitative deificiency_

Disseminated intravascular coagulation (DIC):

  • Widespread activation of coagulation
  • Clotting factors and platelets are consumed → ↑ risk of bleeding
  • Causes: Malignancy, sepsis, trauma, obstetric complications, toxins.
  • Low plts, low fibrinogen, high FDP/ddimer, long PT/INR.
  • Treat the cause and give transfusions, FFP, platelets, cryo etc.

Liver disease:

  • ↓ synthesis of II, V, VII, IX, X, XI and fibrinogen
  • ↓ absorption of vitamin K
  • Abnormalities of platelet function.
How well did you know this?
1
Not at all
2
3
4
5
Perfectly
30
Q

Patient with nephrotic syndrome picture (oedema, proteinuria, low albumin) what test for specific diagnosis?

Options:

24 hour urine

Renal biopsy

A

Renal Biopsy

24hr urine/PCR for Dx of syndrome but Renal Biopsy (in adults, treat for minimal change with steroid in children) is method for Dx

Nephrotic syndrome

Triad of:

  1. Proteinuria (> 3g/24hr) causing
  2. Hypoalbuminaemia (< 30g/L) and
  3. Oedema

Loss of antithrombin-III, proteins C and S and an associated rise in fibrinogen levels predispose to thrombosis. Loss of thyroxine-binding globulin lowers the total, but not free, thyroxine levels

Diagnosis:
Secondary to Systemic Disease

  • DM: glomerulosclerosis
  • SLE: membranous
  • Amyloidosis

Primary:

  • Minimal Change Glomerulonephritis
  • Membranous Nephropathy
  • FSGS
  • Membranoproliferative / Mesangiocapillary GN
How well did you know this?
1
Not at all
2
3
4
5
Perfectly
31
Q

Small cell lung ca, hilar LN, no distant mets - what treatment

Options:

Palliative

Chemo

Radio

Surgery

A

chemotherapy

and radiotherapy​

Lung cancer: small cell

Features

  • usually central
  • arise from APUD* cells
  • associated with ectopic ADH, ACTH secretion
  • ADH → hyponatraemia
  • ACTH → Cushing’s syndrome
  • ACTH secretion can cause bilateral adrenal hyperplasia, the high levels of cortisol can lead to hypokalaemic alkalosis
  • Lambert-Eaton syndrome: antibodies to voltage gated calcium channels causing myasthenic like syndrome

Management

  • usually metastatic disease by time of diagnosis
  • patients with very early stage disease (T1-2a, N0, M0) are now considered for surgery. NICE support this approach in their 2011 guidelines
  • however, most patients with limited disease receive a combination of chemotherapy and radiotherapy
  • patients with more extensive disease are offered palliative chemotherapy

Non-small cell

  1. Squamous cell cancer
    • typically central
    • associated with parathyroid hormone-related protein (PTHrP) secretion → hypercalcaemia
    • strongly associated with finger clubbing
    • hypertrophic pulmonary osteoarthropathy (HPOA)
  2. Adenocarcinoma
    • typically peripheral
    • most common type of lung cancer in non-smokers, although the majority of patients who develop lung adenocarcinoma are smokers
  3. Large cell lung carcinoma
    • typically peripheral
    • anaplastic, poorly differentiated tumours with a poor prognosis
    • may secrete β-hCG

Management

  • only 20% suitable for surgery
  • mediastinoscopy performed prior to surgery as CT does not always show mediastinal lymph node involvement
  • curative or palliative radiotherapy
  • poor response to chemotherapy
  • Surgery contraindications
    • assess general health
    • stage IIIb or IV (i.e. metastases present)
    • FEV1 < 1.5 litres is considered a general cut-off point*
    • malignant pleural effusion
    • tumour near hilum
    • vocal cord paralysis
    • SVC obstruction
How well did you know this?
1
Not at all
2
3
4
5
Perfectly
32
Q

[Repeated number 11]

Legionella - which Abx?

Options:

Ciprofloxacin

Clarithromycin

Cefotaxime

Cefalexin

A

Clarithromycin

How well did you know this?
1
Not at all
2
3
4
5
Perfectly
33
Q

Klebsiella pneumonia → Abx to give.

Options:

Ciprofloxacin

Clarithromycin

Cefotaxime

Co-amoxiclav

A

Cefotaxime

Klebsiella pneumoniae is a Gram-negative rod that is part of the normal gut flora. It can cause a number of infections in humans including pneumonia (typically following aspiration) and urinary tract infections.

Features of Klebsiella pneumonia

  • more common in alcoholic and diabetics
  • may occur following aspiration
  • ‘red-currant jelly’ sputum
  • often affects upper lobes

Prognosis

  • commonly causes lung abscess formation and empyema
  • mortality is 30-50%
How well did you know this?
1
Not at all
2
3
4
5
Perfectly
34
Q

Pneumocystis jiroveci → Abx.

A

Co-Trimoxazole

Pneumocystis jiroveci is an unicellular eukaryote, generally classified as a fungus but some authorities consider it a protozoa

PCP is the most common opportunistic infection in AIDS

all patients with a CD4 count < 200/mm³ should receive PCP prophylaxis

Features

  • dyspnoea
  • dry cough
  • fever
  • very few chest signs

Pneumothorax is a common complication of PCP.
Extrapulmonary manifestations are rare (1-2% of cases), may cause

  • hepatosplenomegaly
  • lymphadenopathy
  • choroid lesions

Investigation

  • CXR: typically shows bilateral interstitial pulmonary infiltrates but can present with other x-ray findings e.g. lobar consolidation. May be normal
  • exercise-induced desaturation
  • sputum often fails to show PCP, bronchoalveolar lavage (BAL) often needed to demonstrate PCP (silver stain shows characteristic cysts)

Management

  • co-trimoxazole
  • IV pentamidine in severe cases
  • steroids if hypoxic (if pO2 < 9.3kPa then steroids reduce risk of respiratory failure by 50% and death by a third)
How well did you know this?
1
Not at all
2
3
4
5
Perfectly
35
Q

Public health question: 200 pts in 2 month period out of 10,000 people get influenza. They ask how many people out of 100,000 in one month period get infection

Option:

100

200

1000

2000

2000

A

1000

How well did you know this?
1
Not at all
2
3
4
5
Perfectly
36
Q

Some guy flew in from Tel Aviv he has chest pain and then a few hours later his leg goes white and there are no pulses.

Options:

Aortic dissection

Thromboses of the popliteal artery

PE

MI

DVT

A

Aortic dissection

nb Chest pain (= acute dissection), followed by acute limb ischaemia (mural thrombus) MI mural thrombus would be more delayed

Associations

  • hypertension
  • trauma
  • bicuspid aortic valve
  • collagens: Marfan’s syndrome, Ehlers-Danlos syndrome
  • Turner’s and Noonan’s syndrome
  • pregnancy
  • syphilis

Features:

  • chest pain: typically severe, radiates through to the back and ‘tearing’ in nature
  • aortic regurgitation
  • hypertension
  • other features may result from the involvement of specific arteries. For example:
    • coronary arteries → angina
    • spinal arteries → paraplegia
    • distal aorta → limb ischaemia
How well did you know this?
1
Not at all
2
3
4
5
Perfectly
37
Q

[repeated]

Patient has bad claudication and can only walk 10m. Distal aorta and both iliac vessels occluded. What do you do?

Options:

Endarterectomy

Femoral crossover bypass

Aorto bifemoral

A

Aorta Bifemoral

Treatment: revascularization typically using either angioplasty or a type of vascular bypass

  • Kissing balloon angioplasty +/- stent, so named because the two common iliacstents touch each other in the distal aorta.
  • Aorto-iliac bypass graft
    • “aorto-bifem” - aortic to both femoral arteries. Used when there is disease at the aortic bifurcation, known as Leriche syndrome, or in both iliac arteries
  • Occasionally:
    • Axillary-bi-femoral and femoral-femoral bypass (sometimes abbreviated “ax-fem fem-fem”)
How well did you know this?
1
Not at all
2
3
4
5
Perfectly
38
Q

[Repeated]

Distal aorta and common iliac disease which bypass do you do?

Options:

Aorta bi-fem

Fem crossover

Iliac endarterectomy

A

Aorta Bifemoral

aortoiliac occlusive disease, aka Leriche’s syndrome, involved blockage of the abdominal aorta as it transitions into the common iliac arteries.

Classic signs/symptoms:

  1. claudication of the buttocks and thighs
  2. absent or decreased femoral pulses
  3. erectile dysfunction

Treatment: revascularization typically using either angioplasty or a type of vascular bypass

  • Kissing balloon angioplasty +/- stent, so named because the two common iliacstents touch each other in the distal aorta.
  • Aorto-iliac bypass graft
    • “aorto-bifem” - aortic to both femoral arteries. Used when there is disease at the aortic bifurcation, known as Leriche syndrome, or in both iliac arteries
  • Axillary-bi-femoral and femoral-femoral bypass (sometimes abbreviated “ax-fem fem-fem”)
How well did you know this?
1
Not at all
2
3
4
5
Perfectly
39
Q

Lady has has surgery 4 days ago, after she was given multiple bags of 5% dextrose. Now she is drowsy. Whats the mechanism?

Options:

Cerebral oedema

Infection

Osmotic demyelination

Osmotic shrinkage

A

Cerebral oedema

Hyponatraemia may be caused by water excess or sodium depletion. Causes of pseudohyponatraemia include hyperlipidaemia (increase in serum volume) or a taking blood from a drip arm. Urinary sodium and osmolarity levels aid making a diagnosis

  • Urinary sodium > 20 mmol/l
    • Sodium depletion, renal loss (patient often hypovolaemic)
      • diuretics
      • Addison’s
      • diuretic stage of renal failure
    • Patient often euvolaemic
      • SIADH (urine osmolality > 500 mmol/kg)
      • hypothyroidism
  • Urinary sodium < 20 mmol/l
    • Sodium depletion, extra-renal loss
      • diarrhoea, vomiting, sweating
      • burns, adenoma of rectum
  • Water excess (patient often hypervolaemic and oedematous)
    • secondary hyperaldosteronism: heart failure, cirrhosis
    • reduced GFR: renal failure
    • IV dextrose, psychogenic polydipsia

Medical emergency:

  • nausea and vomiting (<136 mmol/l)
  • confusion (<131 mmol/l)
  • seizures, non-cardiogenic pulmonary oedema (<125 mmol/l)
  • coma (<117 mmol/l) and eventual death
How well did you know this?
1
Not at all
2
3
4
5
Perfectly
40
Q

60 year old man unilateral left hand tremor at rest. Worse when people looking at tremor. Able to do buttons and hold teacup without shaking. Mild cogwheeling at wrist and fingers on left. What is cause?

Options:

Anxiety state

Early parkinsonism

Benign essential

Right sided cerebellar lesion

A

Early parkinsonism

Parkinson’s disease is a progressive neurodegenerative condition caused by degeneration of dopaminergic neurons in the substantia nigra.. This results in a classic triad of features: bradykinesia, tremor and rigidity. The symptoms of Parkinson’s disease are characteristically asymmetrical.
Epidemiology

  • around twice as common in men
  • mean age of diagnosis is 65 years

Features:

  • Bradykinesia
    • poverty of movement also seen, sometimes referred to as hypokinesia
    • short, shuffling steps with reduced arm swinging
    • difficulty in initiating movement
  • Tremor
    • most marked at rest, 3-5 Hz
    • worse when stressed or tired
    • typically ‘pill-rolling’, i.e. in the thumb and index finger
  • Rigidity
    • lead pipe
    • cogwheel: due to superimposed tremor
  • Other characteristic features
    • mask-like facies
    • flexed posture
    • micrographia
    • drooling of saliva
    • psychiatric features: depression is the most common feature (affects about 40%); dementia, psychosis and sleep disturbances may also occur
    • impaired olfaction
    • REM sleep behaviour disorder
How well did you know this?
1
Not at all
2
3
4
5
Perfectly
41
Q

Stroke with hemiparesis, 2 days later SALT assessment → unsafe swallow. Best feeding?

Options:

NG

PEG

TPN

Thickened fluids

Pureed food

A

NG

  • swollowing may recover, thus, in the early stages a PEG is not indicated. If the patient is determined to have long term swallowing difficulties a PEG may be placed but this would more likely occur at a later date.*
  • Stroke:* Watch the patient swallow a small volume of water; if signs of aspiration (a cough or voice change) make nil by mouth until formal assessment by a speech therapist. Use IV fl uids, then semi-solids (eg jelly; avoid soups and crumbly food). Avoid early NG tube feeds (needed only in the few with established chronic swallowing problems). Speech therapists skilled in assessing swallowing diffi culties are invaluable here.

Enteral feeding

  • Identify patients as malnourished or at risk
    • Eaten nothing or little > 5 days, who are likely to eat little for a further 5 days
    • Poor absorptive capacity
    • High nutrient losses
    • High metabolism
  • Identify unsafe or inadequate oral intake with functional GI tract
  • Consider for enteral feeding
  • Gastric feeding unless upper GI dysfunction (then for duodenal or jejunal tube)
  • Check NG placement using aspiration and pH (check post pyloric tubes with AXR)
  • Gastric feeding > 4 weeks consider long-term gastrostomy
  • Consider bolus or continuous feeding into the stomach
  • ITU patients should have continuous feeding for 16-24h (24h if on insulin)
  • Consider motility agent in ITU or acute patients for delayed gastric emptying. If this doesn’t work then try post pyloric feeding or parenteral feeding.
  • PEG can be used 4 hours after insertion, but should not be removed until >2 weeks after insertion
  • PEG: Percutaneous endoscopic gastrostomy tubes are useful in patients who have a functioning gastrointestinal tract, but where oral feeding is considered unsafe or not possible (e.g. patients with swallowing insuf ficiency following cerebrovascular accident).
How well did you know this?
1
Not at all
2
3
4
5
Perfectly
42
Q

Diagnosing legionella

A

Urinary antigen

legionnaire’s disease is caused by the intracellular bacterium Legionella pneumophilia. It is typically colonizes water tanks and hence questions may hint at air-conditioning systems or foreign holidays. Person-to-person transmission is not seen

Features

  • flu-like symptoms including fever (present in > 95% of patients)
  • dry cough
  • relative bradycardia
  • confusion
  • lymphopaenia
  • hyponatraemia
  • deranged liver function tests
  • pleural effusion: seen in around 30% of patients

Diagnosis: urinary antigen
Management: treat with erythromyci

How well did you know this?
1
Not at all
2
3
4
5
Perfectly
43
Q

Open comminuted fracture with some kind of skin loss and really dirty basically. What do you do?

Options:

External fixation

Internal fixation

Thomas’ splint

Skin traction

Plaster cast

A

Thomas’s Splint : step 1 splint the #

  • May have been remembered incorrectly:*
  • Dirty, compound fracture - Thomas splint*

Comminuted fracture - external fixation; skin infection/soft tissue loss

Paediatric fracture involving the epiphysis requires traction paeds

Open fractures require urgent attention: 6As

  • Analgesia: M+M
  • Assess: NV status, soft tissues, photograph
  • Antisepsis: wound swab, copious irrigation, cover with betadine-soaked dressing.
  • Alignment: align # and splint
  • Anti-tetanus: check status (booster lasts 10yrs)
  • Abx
    • Fluclox 500mg IV/IM + benpen 600mg IV/IM
    • Or, augmentin 1.2g IV

Mx: debridement and fixation in theatre (commonly external fixation to prevent deep seeing of infection)
Gustillo Classification of Open #s
1. Wound <1cm in length
2. Wound ≥1cm c¯ minimal soft tissue damage
3. Extensive soft tissue damage

Clostridium perfringes

  • Most dangerous complication of open #
  • Wound infections and gas gangrene
  • ± shock and renal failure
  • Rx: debride, benpen + clindamycin

Thomas splint:most commonly refers to a splinting device that uses straps attaching over the pelvis or hip as an anchor, a metal rod(s) to mimic normal bone stability and limb length, and a mechanical device to apply traction(used in an attempt to reduce pain, realign the limb, and minimize vascular and neurological complication) to the limb.

A traction splint is commonly used to treat complete long bone fractures of the thigh, femur and not for tibia and fibula area. Their use is common in prehospital care.

How well did you know this?
1
Not at all
2
3
4
5
Perfectly
44
Q

T2DM controlled on diet presenting with red painful eye and blurry vision. What do you do?

Options:

Refer urgently to ophthal

Get urgent retinal imaging

Start topical timolol

Start topical chloramphenicol

Start topical steroids

A

Refer urgently to phthalmology

Glaucoma is a group disorders characterised by optic neuropathy due, in the majority of patients, to raised intraocular pressure (IOP). It is now recognised that a minority of patients with raised IOP do not have glaucoma and vice versa

In acute angle closure glaucoma (AACG) there is a rise in IOP secondary to an impairment of aqueous outflow. Factors predisposing to AACG include:

  • hypermetropia (long-sightedness)
  • pupillary dilatation
  • lens growth associated with age

Features

  • severe pain: may be ocular or headache
  • decreased visual acuity
  • symptoms worse with mydriasis (e.g. watching TV in a dark room)
  • hard, red eye
  • haloes around lights
  • semi-dilated non-reacting pupil
  • corneal oedema results in dull or hazy cornea
  • systemic upset may be seen, such as nausea and vomiting and even abdominal pain

Management

  • urgent referral to an ophthalmologist
  • management options include reducing aqueous secretions with acetazolamide and inducing pupillary constriction with topical pilocarpine
How well did you know this?
1
Not at all
2
3
4
5
Perfectly
45
Q

Guy gets claudication in calf. Where is the block?

Options

SFA

Posterior tibial

Common iliac

A

SFA

  • Calf pain* = superficial femoral disease (commonest)
  • Buttock pain* = iliac disease (internal or common)

Clinical Classification = Fontaine

  1. Asympto (subclinical)
  2. Intermittent claudication
    a. >200m
    b. <200m
  3. Ischaemic rest pain
  4. Ulceration / gangrene
How well did you know this?
1
Not at all
2
3
4
5
Perfectly
46
Q

Guy following RTA has shortened, internally rotated, slightly flexed and adducted right leg. What’s wrong?

Options:

Posterior hip dislocation

Anterior hip dislocation

NOF

Femur shaft fracture

A

Posterior dislocation

Posterior hip dislocation= shortened and internally rotated with flexion and adduction at the hip

Anterior dislocation= externally rotated, abducted, and extended at the hip

NOF= shortened and externally rotated

Femur shaft fracture=shortened with obvious deformity (and potentially low BP)

How well did you know this?
1
Not at all
2
3
4
5
Perfectly
47
Q

Calculate GCS - eyes open to pain, localising to pain, making noises

A

9

Eyes opening response:

  1. No response
  2. To pain
  3. To Speech
  4. Spontaneous

Verbal response:

  1. No response
  2. Incomprehensible sounds
  3. Inappropriate words
  4. Confused
  5. Oriented to time, person, place

Motor Response:

  1. No response
  2. Abnormal extension
  3. Abnormal flexion
  4. Flex to withdraw from pain
  5. Moves to localise pain
  6. Obeys command
How well did you know this?
1
Not at all
2
3
4
5
Perfectly
48
Q

Man does not open his eyes, localises to pain, groaning. What airway is right for him?

Options:

Nasopharyngeal

Oropharyngeal

Cuffed

LMA

Tracheostomy

A

Oropharyngeal

1+5+2=8

GCS 8 => Oropharyngeal

“(GCS) LESS THAN 8, intubate”

Cuffed endotracheal tube

  • Provides optimal control of the airway once cuff inflated
  • May be used for long or short term ventilation
  • Errors in insertion may result in oesophageal intubation (therefore end tidal CO2 usually measured)
  • Paralysis often required
  • Higher ventilation pressures can be used
49
Q

Guy falls 10m, what’s the first thing you do?

A

Secure his airway

??C-Spine first if an option?

50
Q

56 year old with weight loss, heartburn responding to antacids, smoker, drinker. What do you do?

Options:

Gastroscopy

Gastroscopy and treat H pylori if positive

Trial of PPI

More antacids + lifestyle advice

A

Gastroscopy

Investigations for GORD (Isolated symptoms don’t need Ix!)

  • Bloods: FBC
  • CXR: hiatus hernia may be seen
  • OGD if:
    • >55yrs
    • Wt. loss
    • Persistent symptoms despite Rx
    • Dysphagia
    • Symptoms >4wks
  • OGD allows grading by Los Angeles Classification
  • Ba swallow: hiatus hernia, dysmotility
  • 24h pH testing ± manometry
    • pH <4 for >4hrs

Endoscopically proven oesophagitis

  • full dose proton pump inhibitor (PPI) for 1-2 months
  • if response then low dose treatment as required
  • if no response then double-dose PPI for 1 month

Endoscopically negative reflux disease

  • full dose PPI for 1 month
  • if response then offer low dose treatment, possibly on an as-required basis, with a limited number of repeat prescriptions
  • if no response then H2RA or prokinetic for one month

OTC antacids: Gaviscon, Mg trisilicate
1: Full-dose PPI for 1-2mo
Lansoprazole 30mg OD
2: No response → double dose PPI BD
3: No response: add an H2RA
Ranitidine 300mg nocte
Control: low-dose acid suppression PRN

51
Q

Lady with RA and purple nodule on shin ulcerating with raised edges.

Options:

Pyoderma gangrenosum

SCC

venous ulcer

A

Pyoderma Gangrenosum

Features

  • typically on the lower limbs
  • initially small red papule
  • later deep, red, necrotic ulcers with a violaceous border
  • may be accompanied systemic symptoms e.g. Fever, myalgia

Causes

  • idiopathic in 50%
  • inflammatory bowel disease: ulcerative colitis, Crohn’s
  • rheumatoid arthritis, SLE
  • myeloproliferative disorders
  • lymphoma, myeloid leukaemias
  • monoclonal gammopathy (IgA)
  • primary biliary cirrhosis

Management

  • the potential for rapid progression is high in most patients and most doctors advocate oral steroids as first-line treatment
  • other immunosuppressive therapy, for example ciclosporin and infliximab, have a role in difficult cases
52
Q

Guy with prostate ca has back pain and bone scan shows increased uptake in spine, how to help with pain?

Options:

Radiotherapy

Chemotherapy

Bisphosphonates

A

Radiotherapy

metastatic bone pain may respond to strong opioids, bisphosphonates or radiotherapy. The assertion that NSAIDs are particularly effective for metastatic bone pain is not supported by studies. Strong opioids have the lowest number needed to treat for relieving the pain and can provide quick relief, in contrast to radiotherapy and bisphosphonates*. All patients, however, should be considered for referral to a clinical oncologist for consideration of further treatments such as radiotherapy

Most common tumour causing bone metastases (in descending order)

  • prostate
  • breast
  • lung

Most common site (in descending order)

  • spine
  • pelvis
  • ribs
  • skull
  • long bones

??2008 SIGN guidelines: metastatic bone pain may respond to strong opioids, bisphosphonates or radiotherapy. The assertion that NSAIDs are particularly effective for metastatic bone pain is not supported by studies. Strong opioids have the lowest number needed to treat for relieving the pain and can provide quick relief, in contrast to radiotherapy and bisphosphonates*. All patients, however, should be considered for referral to a clinical oncologist for consideration of further treatments such as radiotherapy

53
Q

32 year old woman with mobile non-tender lump in breast.

A

Fibroadenoma

Fibroadenoma:

  • Develop from a whole lobule
  • Mobile, firm breast lumps
  • 12% of all breast masses
  • Over a 2 year period up to 30% will get smaller
  • No increase in risk of malignancy
  • Treatment: If >3cm surgical excision is usual, Phyllodes tumours should be widely excised (mastectomy if the lesion is large)

Breast cysts:

  • Usually presents as a smooth discrete lump (may be fluctuant)
  • Small increased risk of breast cancer (especially if younger)
  • Treatment: Cysts should be aspirated, those which are blood stained or persistently refill should be biopsied or excised

Sclerosing adenosis, (radial scars and complex sclerosing lesions)

  • Usually presents as a breast lump or breast pain
  • Causes mammographic changes which may mimic carcinoma
  • Cause distortion of the distal lobular unit, without hyperplasia (complex lesions will show hyperplasia)
  • Considered a disorder of involution, no increase in malignancy risk
  • Treatment: Lesions should be biopsied, excision is not mandatory

Epithelial hyperplasia

  • Variable clinical presentation ranging from generalised lumpiness through to discrete lump
  • Disorder consists of increased cellularity of terminal lobular unit, atypical features may be present
  • Atypical features and family history of breast cancer confers greatly increased risk of malignanc
  • Treatment: If no atypical features then conservative, those with atypical features require either close monitoring or surgical resection

Fat necrosis

  • Up to 40% cases usually have a traumatic aetiology
  • Physical features usually mimic carcinoma
  • Mass may increase in size initially
  • Treatment: Imaging and core biopsy

Duct papilloma

  • Usually present with nipple discharge
  • Large papillomas may present with a mass
  • The discharge usually originates from a single duct
  • No increase risk of malignancy
  • Treatment: Microdochectomy
54
Q

Some woman with folate and iron deficiency. Some unstated bowel symptoms following bouts of gastroenteritis as a child. What ix do you do?

Options:

Anti TTG abs

Colonoscopy

Schilling’s

A

Anti TTG abs

Coeliac disease is caused by sensitivity to the protein gluten. Repeated exposure leads to villous atrophy which in turn causes malabsorption. Conditions associated with coeliac disease include dermatitis herpetiformis (a vesicular, pruritic skin eruption) and autoimmune disorders (type 1 diabetes mellitus and autoimmune hepatitis).
Diagnosis is made by a combination of immunology and jejunal biopsy. Villous atrophy and immunology normally reverses on a gluten-free diet.
NICE issued guidelines on the investigation of coeliac disease in 2009. If patients are already taking a gluten-free diet they should be asked, if possible, to reintroduce gluten for at least 6 weeks prior to testing.

  • Immunology
    • tissue transglutaminase (TTG) antibodies (IgA) are first-choice according to NICE
    • endomyseal antibody (IgA)
    • anti-gliadin antibody (IgA or IgG) tests are not recommended by NICE
    • anti-casein antibodies are also found in some patients
  • Jejunal biopsy
    • villous atrophy
    • crypt hyperplasia
    • increase in intraepithelial lymphocytes
    • lamina propria infiltration with lymphocytes
  • Rectal gluten challenge has been described but is not widely used

Investigate for coeliac if…

  • signs and symptoms:
    • Chronic or intermittent diarrhoea
    • Failure to thrive or faltering growth (in children)
    • Persistent or unexplained gastrointestinal symptoms including nausea and vomiting
    • Prolonged fatigue (‘tired all the time’)
    • Recurrent abdominal pain, cramping or distension
    • Sudden or unexpected weight loss
    • Unexplained iron-deficiency anaemia, or other unspecified anaemia
  • Conditions:
    • Autoimmune thyroid disease
    • Dermatitis herpetiformis
    • Irritable bowel syndrome
    • Type 1 diabetes
    • First-degree relatives (parents, siblings or children) with coeliac disease

Complications of coeliac disease include:

  • anaemia: iron, folate and vitamin B12 deficiency (folate deficiency is more common than vitamin B12 deficiency in coeliac disease)
  • hyposplenism
  • osteoporosis, osteomalacia
  • lactose intolerance
  • enteropathy-associated T-cell lymphoma of small intestine
  • subfertility, unfavourable pregnancy outcomes
  • rare: oesophageal cancer, other malignancies
55
Q

Post partum breast feeding lady with red tender breast, no mass. What do you do?

Fluclox

Incision and drainage

A

Flucloaxacillin

Mastitis
…affects around 1 in 10 breast feeding women. The BNF advises to treat ‘if systemically unwell, if nipple fissure present, if symptoms do not improve after 12-24 hours of effective milk removal of if culture indicates infection’. The first-line antibiotic is flucloxacillin for 10-14 days. Breast feeding or expressing should continue during treatment.
If left untreated, mastitis may develop into a breast abscess (Red, hot tender swelling). This generally requires incision and drainage.

56
Q

What does a triple assessment consist of?

A

Clinical exam + imaging + core biopsy

<35 USS

>35 Mammography

Cystic = Cytology/FNA

sold = Histology/Core Bx

57
Q

Lady sudden onset severe headache starting 12 hours ago. CT scan normal. What do you do?

Options:

LP

analgesia and review

A

LP

Subarachnoid Haemorrhage

Features

classically: sudden onset severe occipital headache - ‘kicked in the back of the head’

Causes

85% are due to rupture of berry aneurysms (conditions associated with berry aneurysms include adult polycystic kidney disease, Ehlers-Danlos syndrome and coarctation of the aorta)

AV malformations

trauma

tumours

Investigations

CT: negative in 5%

lumbar puncture: done after 12 hrs (allowing time for xanthochromia to develop)

Complications

rebleeding (in 30%)

obstructive hydrocephalus (due to blood in ventricles)

vasospasm leading to cerebral ischaemia

Management

neurosurgical opinion: no clear evidence over early surgical intervention against delayed intervention

post-operative nimodipine (e.g. 60mg / 4 hrly, if BP allows) has been shown to reduce the severity of neurological deficits but doesn’t reduce rebleeding*

58
Q

Old guy with bacteruria grown from catheter. What do you do?

Options:

Treat

Ignore

Treat if symptomatic

Change catheter

A

Treat if symptomatic

59
Q

Guy post thyroidectomy with hoarse voice and weak cough. What damage?

Options:

Recurrent laryngeal

Phrenic nerve

Laryngeal oedema

A

Recurrent laryngeal

Complications following Thyroidectomy

  • Anatomical such as recurrent laryngeal nerve damage.
  • Bleeding. Owing to the confined space haematoma’s may rapidly lead to respiratory compromise owing to laryngeal oedema.
  • Damage to the parathyroid glands resulting in hypocalcaemia.
60
Q

Smoker has lung cancer near right bronchus removed with raised calcium, what will histology show?

Options:

Squamous cell carcinoma

Adenocarcinoma

Small cell carcinoma

Large cell carcinoma

A

Squamous Cell Carcinoma

Squamous cell cancer

  • typically central
  • associated with parathyroid hormone-related protein (PTHrP) secretion → hypercalcaemia
  • strongly associated with finger clubbing
  • hypertrophic pulmonary osteoarthropathy (HPOA)

Adenocarcinoma

  • typically peripheral
  • most common type of lung cancer in non-smokers, although the majority of patients who develop lung adenocarcinoma are smokers

Large cell lung carcinoma

  • typically peripheral
  • anaplastic, poorly differentiated tumours with a poor prognosis
  • may secrete β-hCG

Small Cell

  • usually central
  • arise from APUD* cells
  • associated with ectopic ADH, ACTH secretion
    • ADH → hyponatraemia
    • ACTH → Cushing’s syndrome
    • ACTH secretion can cause bilateral adrenal hyperplasia, the high levels of cortisol can lead to hypokalaemic alkalosis
    • Lambert-Eaton syndrome: antibodies to voltage gated calcium channels causing myasthenic like syndrome
61
Q

Few months breathlessness, fine end-inspiratory bibasal creps, clubbing, JVP at 5cm?

Options:

Pulmonary fibrosis

Congestive cardiac failure

A

Pulmonary fibrosis

Symptoms

  • Dry cough
  • Dyspnoea
  • Malaise, wt. loss
  • Arthralgia
  • OSA

Signs

  • Cyanosis
  • Clubbing
  • Crackles: fine, end-inspiratory

Complications

  • ↑ risk Ca lung
  • Type 2 respiratory failure and cor pulmonale:
    • raised JVP!
62
Q

Man has taken 32 paracetamol 500mg tablets 18 hours ago. He weighs 80kg. You take bloods, but what do you do whilst waiting for the levels to come back?

Options:

Wait for the levels

Give saline

Give naloxone

Give acetylcysteine

A

Give acetylcysteine

…give acetylcysteine as he has taken 200mg/kg, and anything over 150 is probably toxic so should be treated

32x0.5=16g

Single acute overdose is defined as an ingestion of >4 g (or >75 mg/kg) in a period of <1 hour.

->acetylcystein

Management of paracetamol overdoes

  • activated charcoal if ingested < 1 hour ago
  • N-acetylcysteine (NAC)
  • liver transplantation

The following is based on 2012 Commission on Human Medicines (CHM) review of paracetamol overdose management. The big change in these guidelines was the removal of the ‘high-risk’ treatment line on the normogram. All patients are therefore treated the same regardless of risk factors for hepatotoxicity. The National Poisons Information Service/TOXBASE should always be consulted for situations outside of the normal parameters.

Acetylcysteine should be given if:

  • there is a staggered overdose* or there is doubt over the time of paracetamol ingestion, regardless of the plasma paracetamol concentration; or
  • the plasma paracetamol concentration is on or above a single treatment line joining points of 100 mg/L at 4 hours and 15 mg/L at 15 hours, regardless of risk factors of hepatotoxicity

Acetylcysteine is now infused over 1 hour (rather than the previous 15 minutes) to reduce the number of adverse effects.

King’s College Hospital criteria for l_iver transplantation_ (paracetamol liver failure)

  • Arterial pH < 7.3, 24 hours after ingestion
  • or all of the following:
    • prothrombin time > 100 seconds
    • creatinine > 300 µmol/l
    • grade III or IV encephalopathy
63
Q

Causes of collapse

There was one that had an old man collapsing and it said how his wife watched it all and said he didn’t shake and it was only a few seconds but I don’t remember what else happens??

A

Cardiovascular (transient reduction in blood flow to the brain)

  • Arrhythmia: bradycardia (heart block), tachycardia
  • Outflow obstruction: aortic stenosis, Hypertrophic obstructive cardiomyopathy (HOCM), pulmonary embolism, pulmonary stenosis
  • Postural hypotension: hypovolaemia, autonomic neuropathy (e.g. diabetes), antihypertensive medication (e.g. ACE inhibitors)
  • MI, aortic dissection, any condition that reduces cardiac output

Neurological

  • Epilepsy
  • Stroke/transient ischaemic attack (TIA) (rarely)

Vasovagal (reflex bradycardia)

  • Prolonged standing esp. in warm surroundings, emotion
  • Other causes of vagal overactivity: micturition, cough, carotid sinus hypersensitivity (e.g. on shaving the neck or head turning)
  • Metabolic: hypoglycaemia

Note: There is no clearcut loss of consciousness in ‘drop attacks’

64
Q

What is a feature of reactive arthritis other than arthritis, urethritis and uveitis?

A

Keratoderma blenhorrhagicum

Can’t see, can’t pee can’t climb a tree

Reactive arthritis is one of the HLA-B27 associated seronegative spondyloarthropathies. It encompasses Reiter’s syndrome, a term which described a classic triad of urethritis, conjunctivitis and arthritis following a dysenteric illness during the Second World War. Later studies identified patients who developed symptoms following a sexually transmitted infection (post-STI, now sometimes referred to as sexually acquired reactive arthritis, SARA).
Reactive arthritis is defined as an arthritis that develops following an infection where the organism cannot be recovered from the joint.

Features

  • typically develops within 4 weeks of initial infection - symptoms generally last around 4-6 months
  • arthritis is typically an asymmetrical oligoarthritis of lower limbs
  • dactylitis
  • symptoms of urethritis
  • eye: conjunctivitis (seen in 50%), anterior uveitis
  • skin: circinate balanitis (painless vesicles on the coronal margin of the prepuce), keratoderma blenorrhagica (waxy yellow/brown papules on palms and soles)

Around 25% of patients have recurrent episodes whilst 10% of patients develop chronic disease

65
Q

Primary tension pneumo. Management

Options:

Aspirate

Give NIPPV

Give 24% oxygen through venturi

A

Aspiration

Pneumothorax

  • May occur following thoracic trauma when a lung parenchymal flap is created.
  • This acts as a one way valve and allows pressure to rise.
  • The trachea shifts and hyper-resonance is apparent on the affected side.
  • Treatment is with needle decompression (aspiration) and chest tube insertion.
66
Q

Lady with AI history has sore, gritty eye, dry mouth and bilateral parotid swelling?

Options:

Sjogrens

Cancer

Mumps

Sarcoidosis

A

Sjogrens

…. is an autoimmune disorder affecting exocrine glands resulting in dry mucosal surfaces. It may be primary (PSS) or secondary to rheumatoid arthritis or other connective tissue disorders, where it usually develops around 10 years after the initial onset. Sjogren’s syndrome is much more common in females (ratio 9:1). There is a marked increased risk of lymphoid malignancy (40-60 fold)

Features

  • dry eyes: keratoconjunctivitis sicca
  • dry mouth
  • vaginal dryness
  • arthralgia
  • Raynaud’s, myalgia
  • sensory polyneuropathy
  • renal tubular acidosis (usually subclinical)

Investigation

  • rheumatoid factor (RF) positive in nearly 100% of patients
  • ANA positive in 70%
  • anti-Ro (SSA) antibodies in 70% of patients with PSS
  • anti-La (SSB) antibodies in 30% of patients with PSS
  • Schirmer’s test: filter paper near conjunctival sac to measure tear formation
  • histology: focal lymphocytic infiltration
  • also: hypergammaglobulinaemia, low C4

Management

  • artificial saliva and tears
  • pilocarpine may stimulate saliva production
67
Q

Man has no pain, jaundice, dark urine, pale stools?

Options:

Gallstones

Hepatitis

Pancreatic ca

A

Pancreatic cancer

…is often diagnosed late as it tends to present in a non-specific way. Over 80% of pancreatic tumours are adenocarcinomas which typically occur at the head of the pancreas.

Associations

  • increasing age
  • smoking
  • diabetes
  • chronic pancreatitis (alcohol does not appear an independent risk factor though)
  • hereditary non-polyposis colorectal carcinoma
  • multiple endocrine neoplasia
  • BRCA2 gene

Features

  • classically painless jaundice
  • however, patients typically present in a non-specific way with anorexia, weight loss, epigastric pain
  • loss of exocrine function (e.g. steatorrhoea)
  • atypical back pain is often seen
  • migratory thrombophlebitis (Trousseau sign) is more common than with other cancers

Investigation

  • ultrasound has a sensitivity of around 60-90%
  • high resolution CT scanning is the investigation of choice if the diagnosis is suspected

Management

  • less than 20% are suitable for surgery at diagnosis
  • a Whipple’s resection (pancreaticoduodenectomy) is performed for resectable lesions in the head of pancreas. Side-effects of a Whipple’s include dumping syndrome and peptic ulcer disease
  • adjuvant chemotherapy is usually given following surgery
  • ERCP with stenting is often used for palliation
68
Q

Lady has headaches. Transiently loses vision when she strains to defecate. SIgns seen in eyes. She is obese, smokes a lot, drinks a lot, drinks 12 coffees a day. What can she do that will help the most?

Options:

Stop drinking

Stop smoking

Give up coffee

Lose weight

A

Weight loss

Idiopathic intracranial hypertension (also known as pseudotumour cerebri and formerly benign intracranial hypertension) is a condition classically seen in young, overweight females.

Features

  • headache
  • blurred vision
  • papilloedema (usually present)
  • enlarged blind spot
  • sixth nerve palsy may be present

Risk factors

  • obesity
  • female sex
  • pregnancy
  • drugs*: oral contraceptive pill, steroids, tetracycline, vitamin A, lithium

Management

  • weight loss
  • diuretics e.g. acetazolamide
  • topiramate is also used, and has the added benefit of causing weight loss in most patients
  • repeated lumbar puncture
  • surgery: optic nerve sheath decompression and fenestration may be needed to prevent damage to the optic nerve. A lumboperitoneal or ventriculoperitoneal shunt may also be performed to reduce intracranial pressure

*if intracranial hypertension is thought to occur secondary to a known causes (e.g. Medication) then it is of course not idiopathic

69
Q

Man with known AF presents with 1hr hx of epigastric pain 15 minutes after a meal.

Options:

Mesenteric ischaemia

Duodenal ulcer

Biliary colic

Cholestasis

A

Mesenteric Ischaemia

Lower GI ischaemia

Common predisposing factors

  • increasing age
  • atrial fibrillation - particularly for mesenteric ischaemia
  • other causes of emboli: endocarditis, malignancy
  • cardiovascular disease risk factors: smoking, hypertension, diabetes
  • cocaine: ischaemic colitis is sometimes seen in young patients following cocaine use

Common features

  • abdominal pain - in acute mesenteric ischaemia this is often of sudden onset, severe and out-of-keeping with physical exam findings
  • rectal bleeding
  • diarrhoea
  • fever
  • bloods typically show an elevated white blood cell count associated with a lactic acidosis

Diagnosis: CT is the investigation of choice

70
Q

40 year old banker has headaches which have led to taking ibuprofen and paracetamol daily. No neuro or eye signs. No weight loss.

Options:

Advise to stop taking meds and reassess in a month. Add codeine.

CT head

A

Advise to stop taking meds and reassess in a month.

Medication overuse headache

  • Present for 15 days or more per month
  • Developed or worsened whilst taking regular symptomatic medication
  • Patients using opioids and triptans are at most risk
  • May be psychiatric co-morbidity
71
Q

Guy with pneumonia with green sputum, hyponatraemia, euvolaemic. Initial treatment?

Options:

Fluid restrict

Oral vasopressin antagonist

Give normal saline.

A

Fluid restrict

Hyponatraemia:

  • High osmolality
    • Glucose/mannitol infusion
  • Normal osmolality:
    • Spurious
    • Drip arm sample
    • Pseudohyponatraemia (hyperlipidaemia/ paraproteinaemia)
  • Low osmolality
    • True hyponatraemia

True:

  • hypervolaemia
    • CCF, Cirrhosis, Nephrotic syndrome
      • Are they overloaded?
  • euvolaemia
    • Hypothyroidism, adrenal insufficiency, SIADH
      • TFT, short synacthen test, paired urine and serum osmolality
  • Hypovolaemia
    • Diarrhoea, vomiting, diuretics, salt losing nephropathy?
      • need fluid restoring?

SIADH

Causes:

  • Malignancy
    • small cell lung Ca
    • also: pacncreas, prostate
  • Neurological
    • Stroke
    • SAH
    • SDH
    • meningitis, encephalitis, abscess
  • Infections:
    • TB
    • pneumonia
  • Drugs:
    • sulphonylureas
    • SSRIs, tricyclics
    • carbamazepine
    • vincristine
    • cyclophosphamide
  • Other:
    • PEEP
    • porphyrias

Management

  • correction must be done slowly to avoid precipitating central pontine myelinolysis
  • fluid restriction
  • demeclocycline: reduces the responsiveness of the collecting tubule cells to ADH
  • ADH (vasopressin) receptor antagonists have been developed
72
Q

Someone has hearing changes, haemoptysis, nasal discharge, renal problems. CXR shows 3 cavitated lesions?

Wegener’s

Aspergillosis

A

Wegener’s​

Granulomatosis with polyangitis

Signs and symptoms

  • Kidney: rapidly progressive glomerulonephritis (75%), leading to chronic kidney failure
  • Upper airway, eye and ear disease:
    • Nose: pain, stuffiness, nosebleeds, rhinitis, crusting, saddle-nose deformity due to a perforated septum
    • Ears: conductive hearing loss due to auditory tube dysfunction, sensorineural hearing loss (unclear mechanism)
    • Oral cavity: strawberry gingivitis, underlying bone destruction with loosening of teeth, non-specific ulcerations throughout oral mucosa
    • Eyes:pseudotumours, scleritis, conjunctivitis, uveitis, episcleritis
  • Trachea: subglottal stenosis
  • Lungs: pulmonary nodules (referred to as “coin lesions”), infiltrates (often interpreted as pneumonia), cavitary lesions, pulmonary haemorrhage causing haemoptysis, and rarely bronchial stenosis.
  • Arthritis: Pain or swelling (60%), often initially diagnosed as rheumatoid arthritis
  • Skin: nodules on the elbow, purpura, various others (see cutaneous vasculitis)
  • Nervous system: occasionally sensory neuropathy (10%) and rarely mononeuritis multiplex
  • Heart, gastrointestinal tract, brain, other organs: rarely affected.
73
Q

Patient on immunosuppression develops difficulty swallowing, is on lansoprazole, ibuprofen, dexamethasone. Neutropenia. - Cause for dysphagia?

Options:

GORD

Oesophageal candidiasis

A

Oesophageal candidiasis

…. is the most common cause of oesophagitis in patients with HIV. It is generally seen in patients with a CD4 count of less than 100. Typical symptoms include dysphagia and odynophagia. Fluconazole and itraconazole are first-line treatments

GORD primarily presents as dysPEPSIA over dysPHAGIA (difficulty swallowing)

Investigationg of dyspepsia

  • OGD if >55 or ALARMS
  • Try conservative measures for 4 wks
    • Stop drugs: NSAIDs, CCBs (relax LOS)
    • Lose wt., stop smoking, ↓ EtOH
    • Avoid hot drinks and spicy food
    • OTC
      • Antacids: magnesium trisilicate
      • Alginates: gaviscon advance
  • Test for H. pylori if no improvement: breath or serology
    • +ve → eradication therapy
      • Consider OGD if no improvement
    • -ve → PPI trial for 4wks
      • Consider OGD if no improvement
      • PPIs can be used intermittently to control symptoms.
  • Proven GORD
    • Full dose PPI for 1-2mo
    • Then, low-dose PPI PRN
74
Q

Man has low eGFR, high calcium, high creatinine. Calcium was also high a year ago, but not as much?

Options:

Vit D problem

Primary hyperPTH

Secondary hyperPTH

Multiple myeloma

A

Multiple myeloma

Multiple myeloma

Clinical features

  • bone disease: bone pain, osteoporosis + pathological fractures (typically vertebral), osteolytic lesions
  • lethargy
  • infection
  • hypercalcaemia (see below)
  • renal failure
  • other features: amyloidosis e.g. Macroglossia, carpal tunnel syndrome; neuropathy; hyperviscosity

Investigations

  • monoclonal proteins (usually IgG or IgA) in the serum and urine (Bence Jones proteins)
  • increased plasma cells in the bone marrow
  • historically a skeletal survey has been done to look for bone lesions. However, whole-body MRI is increasingly used and is now recommended in the 2016 NICE guidelines

The diagnostic criteria for multiple myeloma requires one major and one minor criteria or three minor criteria in an individual who has signs or symptoms of multiple myeloma.

Major criteria

  • Plasmacytoma (as demonstrated on evaluation of biopsy specimen)
  • 30% plasma cells in a bone marrow sample
  • Elevated levels of M protein in the blood or urine

Minor criteria

  • 10% to 30% plasma cells in a bone marrow sample.
  • Minor elevations in the level of M protein in the blood or urine.
  • Osteolytic lesions (as demonstrated on imaging studies).
  • Low levels of antibodies (not produced by the cancer cells) in the blood.

Hypercalcaemia in myeloma

  • primary factor: due primarily to increased osteoclastic bone resorption caused by local cytokines (e.g. IL-1, tumour necrosis factor) released by the myeloma cells
  • much less common contributing factors: impaired renal function, increased renal tubular calcium reabsorption and elevated PTH-rP levels
75
Q

Guy with lateral epicondyle pain. Which movement hurts?

Wrist extension

Wrist flexion

Elbow flexion

Elbow extension

A

Wrist extension

Lateral epicondylitis typically follows unaccustomed activity such as house painting or playing tennis (‘tennis elbow’). It is most common in people aged 45-55 years and typically affects the dominant arm.

Features

  • pain and tenderness localised to the lateral epicondyle
  • pain worse on wrist extension against resistance with the elbow extended or supination of the forearm with the elbow extended
  • episodes typically last between 6 months and 2 years. Patients tend to have acute pain for 6-12 weeks

Management options

  • advice on avoiding muscle overload
  • simple analgesia
  • steroid injection
  • physiotherapy

Medial epicondylitis=Golfer’s elbow=wrist flexion

76
Q

Guy with dull ache in groin, lump appears on standing, no cough impulse, does not transilluminate…. Cause?

Options:

Varicocele

Inguinal hernia

Epididymal cyst

A

Varicocele

Inguinal hernia COUGH IMPULSE

Epididymal cyst TRANSLUMINATES

Varicocele

  • Palpation
    • Pt. must be standing
    • Can get above mass
    • Separate from testis
    • Feels like a bag of worms
    • Doesn’t transilluminate
    • May have palpable cough impulse
    • Often disappear on lying supine
  • PC: ‘dull ache’
77
Q

49 yo lady with 2 month history of diffuse thyroid swelling and intense sweating at night

Options:

Graves

Toxic nodular goitre

Subacute thyroiditis

A

Graves’ disease

…is the most common cause of thyrotoxicosis. It is typically seen in women aged 30-50 years.

Features

  • typical features of thyrotoxicosis
  • specific signs limited to Grave’s (see below)

Features seen in Graves’ but not in other causes of thyrotoxicosis

  • eye signs (30% of patients): exophthalmos, ophthalmoplegia
  • pretibial myxoedema
  • thyroid acropachy

Autoantibodies

  • TSH receptor stimulating antibodies (90%)
  • anti-thyroid peroxidase antibodies (75%)

Thyrotoxicosis symptoms:

  • Diarrhoea
  • ↑ appetite but ↓ wt.
  • Sweats, heat intolerance
  • Palpitations
  • Tremor
  • Irritability
  • Oligomenorrhoea ± infertility

Signs

  • Hands
    • Fast / irregular pulse
    • Warm, moist skin
    • Fine tremor
    • Palmer erythema
  • Face
    • Thin hair
    • Lid lag
    • Lid retraction
  • Neck
    • Goitre or nodules
78
Q

Bipolar, taking lithium. Glucose normal. Symptoms - thirst, polyuria, hypothyroidism. What is the cause of his symptoms?

Options:

Diabetic insipidus

Diabetes mellitus

Psychogenic polydipsia

Hypothyroidism

A

Diabetic insipidus

Adverse effects of lithium

  • nausea/vomiting, diarrhoea
  • fine tremor
  • nephrotoxicity: polyuria, secondary to nephrogenic diabetes insipidus
  • thyroid enlargement, may lead to hypothyroidism
  • ECG: T wave flattening/inversion
  • weight gain
  • idiopathic intracranial hypertension

Hypothyroidism:

  • Weight gain, lethargy, cold intol
  • skin; dry, cold, yellow, non-pitting ordema, dry coarse hair, loss of lat aspect of eyebrows
  • GI constipation
  • Gynae: menorrhagia
  • Neurological: decreased deep tendon reflexed, carpal tunnel syndrome
79
Q

Test for gestational diabetes.

Options:

Oral glucose tolerance test at 28 weeks

Random blood glucose or sth

Fasting glucose

A

Oral glucose tolerance test at 28 weeks

Screening and diagnosis

  • Screening
    • All women get oral glucose challenge test at 26/40
    • Fasted or fed - 50 mg glucose given, BM taken at 1 hour post-challenge
    • Screen +ve if >7.8 mmol/L
  • Diagnosis
    • 75 mg OGTT at 28/40
    • t = 0, > 6 mmol/L +ve
    • t = 2hr, >7.8 mmol/L +ve
80
Q

Thyroid function low TSH, High T4- post URTI, reduced iodine uptake, hyperthyroid. Cause?

Options:

Viral thyroiditis

Graves

Hashimoto

Nodule

A

Viral thyroiditis

Subacute thyroiditis (also known as De Quervain’s thyroiditis and subacute granulomatous thyroiditis) is thought to occur following viral infection and typically presents with hyperthyroidism.

There are typically 4 phases;

  1. phase 1 (lasts 3-6 weeks): hyperthyroidism, painful goitre, raised ESR
  2. phase 2 (1-3 weeks): euthyroid
  3. phase 3 (weeks - months): hypothyroidism
  4. phase 4: thyroid structure and function goes back to normal

Investigations: globally reduced uptake on iodine-131 scan

Management

  • usually self-limiting - most patients do not require treatment
  • thyroid pain may respond to aspirin or other NSAIDs
  • in more severe cases steroids are used, particularly if hypothyroidism develops
81
Q

Man of 60ish gets diagnosed with diabetes. His BMI is 30. DIet and exercise have failed. What do you do next?

Options:

Metformin

Glicazide

Pioglitazone

SGLT-2 inhibitor

Insulin

A

Metformin

Biguanides – metformin.

Sulfonylureas – gliclazide, glimepiride, glipizide, glyburide.

Thiazolidinediones (Tzd) – pioglitazone, Actos generic.

DPP2 inhibitors - sitagliptin

Incretin mimetics (GLP1 agonist) - exenatide

Alpha-glucosidase inhibitors – Acarbose

SGLT-2 inhibitors - Glioflozin

82
Q

Patient has bowel operation 4 days ago, has not opened bowels for 24 hours. Distended with scant bowel sounds. Cause?

Options:

Post op ileus

Small bowel obstruction

A

Post op ileus

  • due to Fluid sequestration and loss of electrolytes*
  • defined as ileus which persists for more than 3 days following surgery*
83
Q

Patient has operation for perforated duodenal ulcer - gets hiccoughs and nausea - what is the cause?

Options:

Subphrenic abscess

Pelvic abscess

Small bowel obstruction

A

Subphrenic abscess

Subphrenic abscess is a disease characterized by an accumulation of infected fluid between the diaphragm, liver, and spleen. This abscess develops after surgical operations like splenectomy. Presents with cough*, increased *respiratory rate* with shallow respiration, diminished or absent breath sounds, *hiccups*, dullness in percussion, tenderness over the 8th–11th ribs, fever, chills, *anorexia and shoulder tip pain on the affected side. Lack of treatment or misdiagnosis could quickly lead to sepsis, septic shock, and death. It is also associated with peritonitis.

84
Q

Patient with symptoms of guarding, peritonism, what test?

A

Erect CXR

An erect chest x-ray is a useful investigation in patients with an acute abdomen as it may demonstrate free air in the abdomen (pneumoperitoneum) - an abnormal finding suggestive of a perforated abdominal viscus (e.g. a perforated duodenal ulcer).

Rigler’s sign (double wall sign) may be seen on an abdominal film.

CT is now the preferred method for detecting free air in the abdomen.

85
Q

Patient with symptoms of bowel obstruction - what test?

A

Supine AXR

Looking for small and large bowel obstruction is one of the key indications for performing an abdominal film.

  • Small bowel
    • Maximum normal diameter = 35 mm
    • Valvulae conniventes extend all the way across
  • Large bowel
    • Maximum normal diameter = 55 mm
    • Haustra extend about a third of the way across
86
Q

Renal colic - what is the first imaging you’d do?

Options:

AXR

CT KUB

Abdo USS

A

CT KUB

Passmed says: abdominal US but this is referring to the OLD guidelines!

Management:

  • Ureteric calculi less than 5mm
    • Manage expectantly
  • Stone burden of less than 2cm in aggregate
    • Lithotripsy
  • Complex renal calculi and staghorn calculi
    • Percutaneous nephrolithotomy
  • Stone burden of less than 2cm in pregnant females
    • Ureteroscopy
87
Q

Patient has been generally unwell for a week. Now has dysuria and is going frequently. Later on she gets more unwell and has really bad loin pain. What’s wrong?

Options:

Pyelonephritis

Renal colic

A

Pyelonephritis

  • Loin pain
  • Fever and rigors
  • Vomiting
  • If ARF: oliguria
  • dysuria, freq, foul smelling urine
  • +ve urine dip

For patients with sign of acute pyelonephritis hospital admission should be considered local antibiotic guidelines should be followed if available the BNF currently recommends a broad-spectrum cephalosporin or a quinolone (for non-pregnant women) for 10-14 days

88
Q

80 year old with essential hypertension and peripheral vascular disease. What meds

Options:

Diltiazem

Felodipine

Irbesartan

Moxonidine

A

Felodipine

CCB

  • ACEi*= CI in peripheral vascular disease
  • Diltiazem* = (non-DHP calcium channel blocker) caution in patients with bradycardia (avoid if severe); first degree AV block; heart failure; prolonged PR interval; significantly impaired left ventricular function

B-blockers= cause peripheral vasoconstriction + worsen ischaemia in PVD

ARBs= caustion in afro-Caribbean patients—particularly those with left ventricular hypertrophy (may not benefit from an angiotensin-II receptor antagonist); aortic or mitral valve stenosis; elderly (lower initial doses may be appropriate); hypertrophic cardiomyopathy; patients with a history of angioedema; patients with primary aldosteronism (may not benefit from an angiotensin-II receptor antagonist); renal artery stenosis

89
Q

Patient with stiffness in shoulders, thighs, buttocks. fatigue. ESR 80, normal CK.

Options:

Polymyalgia rheumatica

Polymyositis

Dermatomyositis

A

Polymyalgia rheumatica

Pathophysiology

  • overlaps with temporal arteritis
  • histology shows vasculitis with giant cells, characteristically ‘skips’ certain sections of affected artery whilst damaging others
  • muscle bed arteries affected most in polymyalgia rheumatica

Features

  • typically patient > 60 years old
  • usually rapid onset (e.g. < 1 month)
  • aching, morning stiffness in proximal limb muscles (not weakness)
  • also mild polyarthralgia, lethargy, depression, low-grade fever, anorexia, night sweats

Investigations

  • ESR > 40 mm/hr
  • note CK and EMG normal
  • reduced CD8+ T cells

Treatment: prednisolone e.g. 15mg/od - dramatic response

In polymyositis: CK raised and WEAKNESS!

  • Inclusion Body Myositis
  • Asymmetric weakness affecting distal and prox muscles
    • Early involvement of quads, ankle dorsiflexor and wrist/finger flexors
      • → loss of grip strength and ↓dexterity
  • Dysphagia is very common
  • Myalgia is relatively uncommon
90
Q

Testicular pain in a sexually active 18 year old. Management.

A

Take to theatre

Testicular torsion

  • Twist of the spermatic cord resulting in testicular ischaemia and necrosis.
  • Most common in males aged between 10 and 30 (peak incidence 13-15 years)
  • Pain is usually severe and of sudden onset.
  • Cremasteric reflex is lost and elevation of the testis does not ease the pain.
  • Treatment is with surgical exploration. If a torted testis is identified then both testis should be fixed as the condition of bell clapper testis is often bilateral.
91
Q

Someone has AF and has a cold pale leg. It’s been 6 hours.

Options:

Get rid of clot

Amputate

A

Embolectomy

Most likely emboli (AF; mural thrombosis) therefore best method of treatment within 6 hrs is embolectomy

This is an emergency and may require urgent open surgery or
angioplasty. If diagnosis is in doubt, do urgent arteriography. If the occlusion
is embolic, the options are surgical embolectomy (Fogarty catheter) or local
thrombolysis, eg tissue plasminogen activator (t-PA, p339), balancing the risks of surgery with the haemorrhagic complications of thrombolysis. Anticoagulate with heparin after either procedure and look for the source of
emboli. Be aware of possible post-op reperfusion injury and subsequent compartment syndrome.

Surgical: Revascularisation within 6 hours in order to salvage limb. Operative risk is often high due to underlying heart disease. Postoperative anticoagulation is essential.

92
Q

Uvula deviated, cervical lymph nodes, sore throat, inflamed soft palate.

Options:

Peritonsillar abscess

Tonsillitis

Infectious mononucleosis

A

Peritonsillar abscess (quinsy)
… typically develops as a complication of bacterial tonsillitis.

Features include:

  • severe throat pain, which lateralises to one side
  • deviation of the uvula to the unaffected side
  • trismus (difficulty opening the mouth)
  • reduced neck mobility

Patients need urgent review by an ENT speciailist. Most patients are treated with needle aspiration under local anaethaesia.

93
Q

Man had 1cm painless penile ulcer on the shaft. What caused it?

Options:

Neisseria Gonorrhoea

Chlamydia Trachomatis

Treponema Pallidum

Herpes simplex

A

Treponema Pallidum

Painful= Herpes-HSV2 (more commonly) then Chancroid-Haemophilus ducreyi

Painless=Syphilis-Treponema Pallidum (more commonly) then Lymphogranuloma venereum -Chlamydia

  • Genital herpes is most often caused by the herpes simplex virus (HSV) type 2 (cold sores are usually due to HSV type 1). Primary attacks are often severe and associated with fever whilst subsequent attacks are generally less severe and localised to one site
  • Chancroid is a tropical disease caused by Haemophilus ducreyi. It causes painful genital ulcers associated with unilateral, painful inguinal lymph node enlargement. The ulcers typically have a sharply defined, ragged, undermined border.
  • Syphilis is a sexually transmitted infection caused by the spirochaete Treponema pallidum. Infection is characterised by primary, secondary and tertiary stages. A painless ulcer (chancre) is seen in the primary stage. The incubation period= 9-90 days
  • Lymphogranuloma venereum (LGV) is caused by Chlamydia trachomatis. Typically infection comprises of three stages
    • stage 1: small painless pustule which later forms an ulcer
    • stage 2: painful inguinal lymphadenopathy
    • stage 3: proctocolitis

LGV is treated using doxycycline.

Other causes of genital ulcers:

  • Behcet’s disease
  • carcinoma
  • granuloma inguinale: Klebsiella granulomatis
94
Q

Lady with flushing, pustular rash, telangectasia on her face.

Options:

Rosacea

Lupus pernio

Acne

Discoid lupus

A

Acne rosacea

…is a chronic skin disease of unknown aetiology

Features

  • typically affects nose, cheeks and forehead
  • flushing is often first symptom
  • telangiectasia are common
  • later develops into persistent erythema with papules and pustules
  • rhinophyma
  • ocular involvement: blepharitis

Management

  • topical metronidazole may be used for mild symptoms (i.e. Limited number of papules and pustules, no plaques)
  • more severe disease is treated with systemic antibiotics e.g. Oxytetracycline

recommend daily application of a high-factor sunscreen

camouflage creams may help conceal redness

laser therapy may be appropriate for patients with prominent telangiectasia

95
Q

Gentlemen with bald head, lumpy hyperkeratotic lesions on scalp. What is he at risk of?

A

SCC

Risk factors include:

  • excessive exposure to sunlight / psoralen UVA therapy
  • actinic keratoses and Bowen’s disease
  • immunosuppression e.g. following renal transplant, HIV
  • smoking
  • long-standing leg ulcers (Marjolin’s ulcer)
  • genetic conditions e.g. xeroderma pigmentosum, oculocutaneous albinism

Treatment: Surgical excision with 4mm margins if lesion <20mm in diameter. If tumour >20mm then margins should be 6mm. Mohs micrographic surgery may be used in high-risk patients and in cosmetically important sites.

Actinic, or solar, keratoses (AK) is a common premalignant skin lesion that develops as a consequence of chronic sun exposure

  • Features
    • small, crusty or scaly, lesions
    • may be pink, red, brown or the same colour as the skin
    • typically on sun-exposed areas e.g. temples of head
    • multiple lesions may be present
  • Management options include
    • prevention of further risk: e.g. sun avoidance, sun cream
    • fluorouracil cream: typically a 2 to 3 week course. The skin will become red and inflamed - sometimes topical hydrocortisone is given following fluorouracil to help settle the inflammation
    • topical diclofenac: may be used for mild AKs. Moderate efficacy but much fewer side-effects
    • topical imiquimod: trials have shown good efficacy
    • cryotherapy
    • curettage and cautery
96
Q

Lesion on shin, irregular borders, multiple colours. You suspect melanoma. What’s the commonest type?

A

Superficial spreading

Classification of Malignant Melanoma

  • Superficial Spreading: 80%
    • Irregular boarders, colour variation
    • Commonest in Caucasians
    • Grow slowly, metastasise late = better prognosis
  • Lentigo Maligna Melanoma
    • Often elderly pts.
    • Face or scalp
  • Acral Lentiginous
    • Asians/blacks
    • Palms, soles, subungual (c¯ Hutchinson’s sign)
  • Nodular Melanoma
    • All sites
    • Younger age, new lesion
    • Invade deeply and metastasis early = poor prog
  • Amelanotic
    • Atypical appearance → delayed Dx

Staging and Prognosis

  • Breslow Depth
    • Thickness of tumour to deepest point of dermal invasion
    • <1mm = 95-100% 5ys
    • >4mm = 50% 5ys
  • Clark’s Staging
    • Stratifies depth by 5 anatomical levels
    • Stage 1: Epidermis
    • Stage 5: sc fat

nb metastasis to liver/eye!

97
Q

Guy with pale stools, abdo pain. Xray shows area of calcification.

Diagnosis?

A

Dx - decrease in exocrine pancreatic function

Chronic pancreatitis…is an inflammatory condition which can ultimately affect both the exocrine and endocrine functions of the pancreas. Around 80% of cases are due to alcohol excess with up to 20% of cases being unexplained

  • Features
    • pain is typically worse 15 to 30 minutes following a meal
    • steatorrhoea: symptoms of pancreatic insufficiency usually develop between 5 and 25 years after the onset of pain
    • diabetes mellitus develops in the majority of patients. It typically occurs more than 20 years after symptom begin
  • Investigation
    • abdominal x-ray shows pancreatic calcification in 30% of cases
    • CT is more sensitive at detecting pancreatic calcification. Sensitivity is 80%, specificity is 85%
    • functional tests: faecal elastase may be used to assess exocrine function if imaging inconclusive
  • Management
    • pancreatic enzyme supplements
    • analgesia
    • antioxidants: limited evidence base - one study suggests benefit in early disease
98
Q

Lady with #NOF - IV paracetamol not helping. What analgesia next?

Options:

Patient controlled pethidine

Fmoral block

IM diclofenac

IV repeated bolus of morphine

Gas and Air

A

Femoral nerve block

Paracetamol, not working

NSAIDs CI

opiods: IV repeated bolus of morphine

Patient controlled pethidine: used post op for major abdominal surgery

  • 1.3 Analgesia*
  • 1.3.1Assess the patient’s pain:*
  • immediately upon presentation at hospital and*
  • within 30 minutes of administering initial analgesia and hourly until settled on the ward and*
  • regularly as part of routine nursing observations throughout admission. [2011]*
  • 1.3.2Offer immediate analgesia to patients presenting at hospital with suspected hip fracture, including people with cognitive impairment. [2011]*
  • 1.3.3Ensure analgesia is sufficient to allow movements necessary for investigations (as indicated by the ability to tolerate passive external rotation of the leg), and for nursing care and rehabilitation. [2011]*
  • 1.3.4Offer paracetamol every 6 hours preoperatively unless contraindicated. [2011]*
  • 1.3.5Offer additional opioids if paracetamol alone does not provide sufficient preoperative pain relief. [2011]*
  • 1.3.6Consider adding nerve blocks if paracetamol and opioids do not provide sufficient preoperative pain relief, or to limit opioid dosage. Nerve blocks should be administered by trained personnel. Do not use nerve blocks as a substitute for early surgery. [2011]*
  • 1.3.7Offer paracetamol every 6 hours postoperatively unless contraindicated. [2011]*
  • 1.3.8Offer additional opioids if paracetamol alone does not provide sufficient postoperative pain relief. [2011]*
  • 1.3.9Non-steroidal anti-inflammatory drugs (NSAIDs) are not recommended. [2011]*
99
Q

Guy who was lifting heavy things. Now just lumbar pain.

Options:

Symptom control with NSAID

Emergency MRI

A

Symptom control with NSAID

Management of musculoskeletal AND prolapsed disc:

  • analgesia, physiotherapy, exercises
  • if symptoms persist then referral for consideration of MRI is appropriate

Investigation

  • lumbar spine x-ray should not be offered
  • MRI should only be offered to patients with non-specific back pain ‘only if the result is likely to change management’ and to patients where malignancy, infection, fracture, cauda equina or ankylosing spondylitis is suspected

Advice to people with low back pain

  • try to encourage self-management
  • stay physically active and exercise
  • Analgesia
    • NSAIDS are now recommended first-line for patients with back pain. This follows studies that show paracetamol monotherapy is relatively ineffective for back pain
    • proton pump inhibitors should be co-prescribed for patients over the age of 45 years who are given NSAIDs
    • NICE guidelines on neuropathic pain should be followed for patients with sciatica

Red flags for lower back pain

  • age < 20 years or > 50 years
  • history of previous malignancy
  • night pain
  • history of trauma
  • systemically unwell e.g. weight loss, fever
100
Q

Hayfever symptoms, how is it mediated?

Options:

IgA, IgE, IgG, IgM, IgD

A

IgE

Allergic rhinitis

The underlying mechanism involves IgE antibodies attaching to the allergen and causing the release of inflammatory chemicals such as histamine from mast cells. Diagnosis is usually based on a medical history in combination with a skin prick test or blood tests for allergen-specific IgE antibodies. These tests, however, are sometimes falsely positive. The symptoms of allergies resemble those of the common cold; however, they often last for more than two weeks and typically do not include a fever.

101
Q

Features of AS:

A

slow rising pulse

Symptoms

  • Triad: angina, dyspnoea, syncope (esp. c¯ exercise)
  • LVF: PND, orthopnoea, frothy sputum
  • Arrhythmias
  • Systemic emboli if endocarditis
  • Sudden death

Signs

  • Slow rising pulse c¯ narrow PP
  • Aortic thrill
  • Apex: Forceful, non-displaced (pressure overload)
  • Heart Sounds
    • Quiet A2
    • Early syst. ejection click if pliable (young) valve
    • S4 (forceful A contraction vs. hypertrophied V)
  • Murmur
    • ESM
    • Right 2nd ICS
    • Sitting forward in end-expiration
    • Radiates to carotids

Clinical Indicators of Severe AS

  • Quiet / absent A2
  • S4
  • Narrow pulse pressure
  • Decompensation: LVF
102
Q

Patient with 3 weeks of fevers, cough, feeling unwell. On exam: dull percussion, absent VR, reduced expansion

Options:

Empyema

Consolidation
Collapse

A

Empyema

Presentation

  • Symptoms
    • Asymptomatic
    • Dyspnoea
    • Pleuritic chest pain
  • Signs
    • Tracheal deviation away from effusion
    • ↓ expansion
    • Stony dull percussion
    • ↓ air entry
    • Bronchial breathing just above effusion
    • ↓VR

Empyema=Pus in the pleural cavity

  • Anaerobes, Staph, Gm-ve
  • Assoc. c¯ recurrent aspiration
  • Pt. c¯ resolving pneumonia develops recurrent fever
  • Tap: turbid, pH<7.2, ↓glucose, ↑LDH
  • Mx: US guided chest drain + Abx
103
Q

What vitamin you have to give somebody when starting TB treatment?

A

Pyridoxine

Isoniazid

  • mechanism of action: inhibits mycolic acid synthesis
  • peripheral neuropathy: prevent with pyridoxine (Vitamin B6)
  • hepatitis, agranulocytosis
  • liver enzyme inhibitor

Rifampicin

  • mechanism of action: inhibits bacterial DNA dependent RNA polymerase preventing transcription of DNA into mRNA
  • potent liver enzyme inducer
  • hepatitis, orange secretions
  • flu-like symptoms

Pyrazinamide

  • mechanism of action: converted by pyrazinamidase into pyrazinoic acid which in turn inhibits fatty acid synthase (FAS) I
  • hyperuricaemia causing gout
  • arthralgia, myalgia
  • hepatitis

Ethambutol

  • mechanism of action: inhibits the enzyme arabinosyl transferase which polymerizes arabinose into arabinan
  • optic neuritis: check visual acuity before and during treatment
  • dose needs adjusting in patients with renal impairment
104
Q

Somebody comes in to A&E with drug resistant TB, where do you keep them?

Options:

Side room + respiratory measures for staff

Negative pressure room + respiratory measures for staff

A

Negative pressure room + respiratory measures for staff

  • Isolate patient in a side room with negative pressure ventilation or ventilated to an outside window
  • Wear gowns and masks if the patient has multi-drug-resistant TB (or when performing aerosol-generating procedures if non MDR-TB)
  • Ensure the patient wears a mask if moving outside the side room
105
Q

Somebody with feautres of UC - how do you investigate

Options:

Flexi sigmoidoscopy

CT

US

A

Flexi sigmoidoscopy

~Investigations:

  • FBC - anaemia due to blood loss; leukocytosis
  • ESR - increased; correlates with active disease
  • CRP - raised; but less so than in Crohn’s disease
  • biochemistry - in active disease, biochemical abnormalities may include hypokalaemia, hyponatraemia, hypomagnesaemia, hypocalcaemia, and hyoalbuminaemia. Abnormal LFTs due to associated chronic active hepatitis - increased ALT - or sclerosing cholangitis - increased alkaline phosphatase
  • ANCA - found in HLA-DR2 associated form of ulcerative colitis
  • radiology:
    • plain abdominal x-ray - excludes toxic dilatation, which is more than 5.5 cm in diameter in adults
    • barium enema:
      • diagnosis of extent and severity of the disease
      • procedure is contraindicated in those patients at risk of a toxic dilatation
  • rectal biopsy - taken at sigmoidoscopy
  • colonoscopy - this is contraindicated in those patients at risk of toxic dilatation. Allows multiple biopsies to be taken throughout the colon and delineation of the extent and activity of the disease
  • white cell scan - allows imaging in severe disease
  • molecular biology - a high intensity of CD44v6 and v3 epitope expression on crypt epithelial cells in patients with UC has been noted. This observation may have diagnostic potential in distinguishing UC from Crohn’s

Diagnosis:

  • proctosigmoidoscopy or colonoscopy with characteristic changes of ulcerative colitis (loss of the typical vascular pattern, granularity, friability, and ulceration)
  • biopsy - to distinguish UC from infectious colitis
  • negative stool examination - for infectious causes
106
Q

Pt comes in fitting for ages, given two loads of lorazepam, whats next?

A

Phenytoin loading

This is a medical emergency. The priority is termination of seizure activity, which if prolonged will lead to irreversible brain damage. First-line drugs are benzodiazepines such as diazepam or lorazepam. If ineffective within 10 minutes it is appropriate to start a second-line agent such as phenytoin, sodium valproate, levetiracetam, or phenobarbital. If no response within 30 minutes from onset, then the best way to achieve rapid control of seizure activity is induction of general anaesthesia.

107
Q

Lady with splenectomy, needs to be on penicillin - why?

Options:

Pneumococcus

Staph aureus

Haemophilus

A

Pneumococcus

The increased risk of infection in patients with hyposplenism is lifelong, but is highest early after splenectomy, the highest risk being from pneumococcal infection.

Hyposplenism may complicate certain medical conditions where splenic atrophy occurs or may be the result of medical intervention such as splenic artery embolization and splenectomy for trauma. Diagnosis of hyposplenism is difficult and whilst there may be peripheral markers of the splenectomised state (e.g. Howell-Jolly bodies) these are neither 100% sensitive or specific. The most sensitive test is a radionucleotide labelled red cell scan.

Hyposplenism, by whatever mechanism it occurs dramatically increases the risk of post-splenectomy sepsis, particularly with encapsulated organisms. Since these organisms may be opsonised, but this then goes undetected at an immunological level due to loss of the spleen. For this reason, individuals are recommended to be vaccinated and have antibiotic prophylaxis.

Key recommendations

  • All those with hyposplenism or may become so (such as prior to an elective splenectomy) should receive pneumococcal, Haemophilus type b and meningococcal type C vaccines. These should be administered 2 weeks prior to splenectomy or two weeks following splenectomy. The vaccine schedule for meningococcal disease essentially consists of a dose of Men C and Hib at 2 weeks and then a dose of the MenACWY vaccine one month later. Those aged under 2 may require a booster at 2 years. A dose of pneumococcal polyvalent polysaccharide vaccine (PPV) is given at two weeks. A conjugated vaccine (PCV) is offered to young children. The PCV is more immunogenic but covers fewer serotypes. Boosting PPV is either guided by serological measurements (where available) or by routine boosting doses at 5 yearly intervals.
  • Annual influenza vaccination is recommended in all cases
  • Antibiotic prophylaxis is offered to all. The risk of post-splenectomy sepsis is greatest immediately following splenectomy and in those aged less than 16 years or greater than 50 years. Individuals with a poor response to pneumococcal vaccination are another high-risk group. High-risk individuals should be counselled to take penicillin or macrolide prophylaxis. Those at low risk may choose to discontinue therapy. All patients should be advised about taking antibiotics early in the case of intercurrent infections.
  • Asplenic individuals travelling to malaria endemic areas are at high risk and should have both pharmacological and mechanical protection.
108
Q

Pt with known renal cell cancer now gets swollen legs and distended veins below umbilicus what’s the diagnosis?

Options:

Obstruction of IVC

Lymphatics

A

Obstruction of IVC

IVCS is a result of obstruction of the inferior vena cava.

Presentation

  • Edema of the lower extremities (peripheral edema), caused by an increase in the blood pressure in the veins.
  • Tachycardia. This is caused by the decreased preload, causing the heart to increase its frequency.
  • In pregnant women, signs of fetal hypoxia and distress may be seen in the cardiotocography. This is caused by decreased perfusion of the uterus, resulting in hypoxemia of the fetus.
  • Supine hypotensive syndrome

Causes

  • Obstruction by deep vein thrombosis or tumors (most commonly renal cell carcinoma)
  • Compression through external pressure by neighbouring structures or tumors, either by significantly compressing the vein or by promoting thrombosis by causing turbulence by disturbing the blood flow. This is quite common during the third trimester of pregnancy when the uterus compresses the vein in the right side position.
  • Iatrogenic causes may be suspected in patients with a medical history of liver transplantion, vascular catheters, dialysis and other invasive procedures in the vicinity
  • Budd-Chiari syndrome
109
Q

Some blood gas to interpret

A
  1. How is the patient?
  2. Is the patient hypoxaemic? the Pa02 on air should be >10 kPa
  3. Is the patient acidaemic (pH <7.35) or alkalaemic (pH >7.45)
  4. Respiratory component: What has happened to the PaCO2?
  • PaCO2 > 6.0 kPa suggests a respiratory acidosis (or respiratory compensation for a metabolic alkalosis)
  • PaCO2 < 4.7 kPa suggests a respiratory alkalosis (or respiratory compensation for a metabolic acidosis)
  1. Metabolic component: What is the bicarbonate level/base excess?
  • bicarbonate < 22 mmol/l (or a base excess < - 2mmol/l) suggests a metabolic acidosis (or renal compensation for a respiratory alkalosis)
  • bicarbonate > 26 mmol/l (or a base excess > + 2mmol/l) suggests a metabolic alkalosis (or renal compensation for a respiratory
110
Q

Patient with severe pain on defication and blood on paper:

Options:

Fissure

Rectal ca

Haemoorhoids

A

Anal fissure

…longitudinal or elliptical tears of the squamous lining of the distal anal canal. If present for less than 6 weeks they are defined as acute, and chronic if present for more than 6 weeks. Around 90% of anal fissures occur on the posterior midline

Risk factors

  • constipation
  • inflammatory bowel disease
  • sexually transmitted infections e.g. HIV, syphilis, herpes

Features painful, bright red, rectal bleeding
Management of an acute anal fissure (< 6 weeks)

  • dietary advice: high-fibre diet with high fluid intake
  • bulk-forming laxatives are first line - if not tolerated then lactulose should be tried
  • lubricants such as petroleum jelly may be tried before defecation
  • topical anaesthetics
  • analgesia
    • topical steroids do not provide significant relief

Management of a chronic anal fissure (> 6 weeks)

  • the above techniques should be continued
  • topical glyceryl trinitrate (GTN) is first line treatment for a chronic anal fissure
  • if topical GTN is not effective after 8 weeks then secondary referral should be considered for surgery or botulinum toxin
111
Q

Patient ventilated on ITU, develops respitatory alkalosis, what do you do?

Options:

Decrease frequency of ventilations

Decrease dead space

A

Decrease frequency of ventilations

->CO2 retention->acidosis

Respiratory alkalosis occurs when an increase in ventilatory volume and/or rate causes a decrease in PaCO2. Such increased ventilation is often in response to pain, anxiety, hypoxia or fever – or when the patient on mechanical mandatory ventilation is ‘over-ventilated’.

Metabolic alkalosis is characterised by an increase in bicarbonate with or without a compensatory increase in CO2. It may occur from:

  • Excess acid loss (such as in pyloric stenosis).
  • Excess ingestion of alkali (rare).
  • Renal bicarbonate retention (rare).
  • As a consequence of hypokalaemia (causing a shift in H+).
112
Q

Gives FEV1/FEVC ratio - shows restrictive pattern. Dx?

Options:

Pulmonary fibrosis

COPD

Bronchiectasis

A

Pulmonary fibrosis

Obstructive lung disease

  • Spirometry
    • FEV1 - significantly reduced
    • FVC - reduced or normal
    • FEV1% (FEV1/FVC) - reduced
  • DDx
    • Asthma
    • COPD
    • Bronchiectasis
    • Bronchiolitis obliterans

Restrictive lung disease

  • spirometry
    • FEV1 - reduced
    • FVC - significantly reduced
    • FEV1% (FEV1/FVC) - normal or increased
  • DDx
    • Pulmonary fibrosis
    • Asbestosis
    • Sarcoidosis
    • Acute respiratory distress syndrome
    • Infant respiratory distress syndrome
    • Kyphoscoliosis
    • Neuromuscular disorders
113
Q

Some guy with obstructive features, long term smoker. Dx?

Options:

Chronic bronchtis

Pulmonary fibrosis

A

Chronic Bronchitis

cough and sputum production on most days for 3mo of 2 successive years.

  • Obstructive lung disease
    • Spirometry
      • FEV1 - significantly reduced
      • FVC - reduced or normal
      • FEV1% (FEV1/FVC) - reduced
    • DDx
      • Asthma
      • COPD
      • Bronchiectasis
      • Bronchiolitis obliterans
  • Restrictive lung disease
    • spirometry
      • FEV1 - reduced
      • FVC - significantly reduced
      • FEV1% (FEV1/FVC) - normal or increased
    • DDx
      • Pulmonary fibrosis
      • Asbestosis
      • Sarcoidosis
      • Acute respiratory distress syndrome
      • Infant respiratory distress syndrome
      • Kyphoscoliosis
      • Neuromuscular disorders
114
Q

Guy who is IVDU develops harsh murmur. What’s the cause?

Options:

Staph Aureus

Strep epidermidis

A

Staph Aureus

The following types of patients are affected:

  • previously normal valves (50%, typically acute presentation)
  • rheumatic valve disease (30%)
  • prosthetic valves
  • congenital heart defects
  • intravenous drug users (IVDUs, e.g. typically causing tricuspid lesion)

Causes

  • historically Streptococcus viridans was the most common cause of infective endocarditis. This is no longer the case, except in developing countries. Staphylococcus aureus is now the most common cause of infective endocarditis. Staphylococcus aureus is also particularly common in acute presentation and IVDUs
  • coagulase-negative Staphylococci such as Staphylococcus epidermidis commonly colonize indwelling lines and are the most cause of endocarditis in patients following prosthetic valve surgery, usually the result of perioperative contamination. After 2 months the spectrum of organisms which cause endocarditis return to normal (i.e. Staphylococcus aureus is the most common cause)
  • Streptococcus viridans still accounts for around 20% of cases. Technically Streptococcus viridans is a pseudotaxonomic term, referring to viridans streptococci, rather than a particular organism. The two most notable viridans streptococci are Streptococcus mitis and Streptococcus sanguinis. They are both commonly found in the mouth and in particular dental plaque so endocarditis caused by these organisms is linked with poor dental hygiene or following a dental procedure
  • Streptococcus bovis is associated with colorectal cancer
  • non-infective: systemic lupus erythematosus (Libman-Sacks), malignancy: marantic endocarditis

Culture negative causes:

  • prior antibiotic therapy
  • Coxiella burnetii
  • Bartonella
  • Brucella
  • HACEK: Haemophilus, Actinobacillus, Cardiobacterium, Eikenella, Kingella)
115
Q

20 year old female, walks in, has 2yr history of intermittent diarrhoea, IDA, aphthous ulcers, raised ESR -

Options:

Crohns

Coeliac

A

Crohn’s

…is a form of inflammatory bowel disease. It commonly affects the terminal ileum and colon but may be seen anywhere from the mouth to anus.

Pathology

  • cause is unknown but there is a strong genetic susceptibility
  • inflammation occurs in all layers, down to the serosa. This is why patients with Crohn’s are prone to strictures, fistulas and adhesions

Crohn’s disease typically presents in late adolescence or early adulthood. Features include:

  • presentation may be non-specific symptoms such as weight loss and lethargy
  • diarrhoea: the most prominent symptom in adults. Crohn’s colitis may cause bloody diarrhoea
  • abdominal pain: the most prominent symptom in children
  • perianal disease: e.g. Skin tags or ulcers
  • extra-intestinal features are more common in patients with colitis or perianal disease

Investigations

  • raised inflammatory markers
  • increased faecal calprotectin
  • anaemia
  • low vitamin B12 and vitamin D
116
Q

2 days post-MI, pt develops SOB, raised JVP and harsh systolic murmur.

A

Mitral regurgitation

  • MI->Papillary muscle / chordae → MR
    • (PSM + Pulmonary oedema)
  • MI-> Septum rupture
    • (PSM + ↑JVP + Heart failure)
  • Mi-> Left ventricular free wall rupture ->Cardiac tamponade
    • (Beck’s triad (↓BP, ↑JVP, muffled heart sounds) + Pulsus paradoxus)

Mitral Regurgitation:

Causes

  • Mitral valve prolapse
  • LV dilatation: AR, AS, HTN
  • Annular calcification → contraction (elderly)
  • Post-MI: papillary muscle dysfunction/rupture
  • Rheumatic fever
  • Connective tissue: Marfan’s, Ehlers-Danlos

Symptoms

  • Dyspnoea, fatigue
  • AF → palpitations + emboli
  • Pulmonary congestion → HTN + oedema

Signs

  • AF
  • Left parasternal heave (RVH)
  • Apex: displaced
    • Volume overload as ventricle has to pump forward SV and regurgitant volume
    • -> eccentric hypertrophy
  • Heart Sounds
    • Soft S1
    • S2 not heard separately from murmur
    • Loud P2 (if PTH)
  • Murmur
    • Blowing PSM
    • Apex
    • Left lateral position in end expiration
    • Radiates to the axilla
117
Q

A man with stab injury to the back - hemisection of the cord. Where would he lose pain senstion from?

A

Brown-Sequard syndrome (spinal cord hemisection)

Tracts affected:

  1. Lateral corticospinal tract
  2. Dorsal columns
  3. Lateral spinothalamic tract

Clinical notes

  1. Ipsilateral spastic paresis below lesion
  2. Ipsilateral loss of proprioception and vibration sensation
  3. Contralateral loss of pain and temperature sensation
118
Q

A ECG shows STEMI in leads V4-V6 and I, AvL - where is the infarct?

Options:

Antero-lateral

Antero-septal

Inferior

A

Antero-lateral

=Circumflex artery